patho2

Pataasin ang iyong marka sa homework at exams ngayon gamit ang Quizwiz!

A 29-year-old construction worker got a sliver under his fingernail 4 days ago. The affected finger is now reddened, painful, swollen, and warm to touch. Which of the following hematological processes is most likely occurring in the bone marrow in response to the infection? A)Proliferation of immature neutrophils B)High circulatory levels of myeloblasts C)Increased segmented neutrophil production D)Phagocytosis by myelocytes

A Feedback: A part of the immune response is the proliferation of neutrophils in response to infectious organisms. Neutrophilia is an increase in immature neutrophils ("bands") most commonly seen in acute infections and tissue injuries that promote the accelerated release of neutrophils and their precursors into the circulation. Myeloblasts are not normally found in circulation, and segmented neutrophils would not yet have reached maturity (takes approximately 2 weeks to mature). Myelocytes do not perform phagocytosis.

Which of the following assessments and laboratory findings would be most closely associated with acute leukemia? A)High blast cell counts and fever B)Decreased oxygen partial pressure and weight loss C)Increased serum potassium and sodium levels D)Increased blood urea nitrogen and bone pain

A Feedback: Acute leukemia is often marked by a fever as well as leukostasis. Changes in oxygen saturation, electrolytes, and BUN would be less closely associated with ALL and AML.

The nurse knows high incidences of infectious illnesses among the older adults who reside in a long-term care facility are most likely to have diminished immune capacity because of A) decreased numbers and responsiveness of T lymphocytes. B) decreased antigen recognition by B lymphocytes. C) overexpression of cytokines and receptors. D) altered function in peripheral lymphocytes.

A Feedback: Although this phenomenon is not well understood, increasing proportions of lymphocytes become unresponsive with age, and CD4+ T lymphocytes are the most severely affected. B lymphocytes recognize more antigens, not fewer, and expression of cytokines and their cellular receptors decreases.

Which of the following scenarios best describes an example of infection originating with a fomite? A) A client who contracted hepatitis C through sharing a contaminated syringe with an infected person B) A nurse with a positive tuberculin screening test (PPD) after admitting a patient diagnosed with tuberculosis (TB) C) A woman who contracted Lyme disease after a tick bite D) A man who has been diagnosed with trichinosis after eating undercooked pork

A Feedback: An object that carries an infectious organism, such as a dirty syringe, is known as a fomite. Airborne contact with TB does not utilize a fomite. An infection via contact with an infected arthropod constitutes zoonosis. A person who consumes ova in undercooked meat acquires an infection by ingestion.

When discussing the sequence of clot dissolution, the science instructor will talk about which item that begins the process? A)Plasminogen B)Dabigatran C)Platelets D)á2-plasmin inhibitor

A Feedback: As with clot formation, clot dissolution requires a sequence of steps controlled by activators and inhibitors. Plasminogen, the proenzyme for the fibrinolytic process, normally is present in the blood in its inactive form. It is converted to its active form, plasmin, by plasminogen activators formed in the vascular endothelium, liver, and kidneys. Dabigatran is an anticoagulant. Platelets actually help cells stick together or adhere.

Misinterpreting her physician's instructions, a 69-year-old woman with a history of peripheral artery disease has been taking two 325 mg tablets of aspirin daily. How has this most likely affected her hemostatic status? A) Irreversible acetylation of platelet cyclooxygenase activity has occurred. B) The patient's prostaglandin (TXA2) levels are abnormally high. C) She is at risk of developing secondary immune thrombocytopenic purpura (ITP). D) The binding of an antibody to platelet factor IV produces immune complexes.

A Feedback: Aspirin can cause inhibition of platelet aggregation that lasts for the life of the platelet. High TXA2 levels would be associated with increased coagulability. ITP would not result from aspirin intake, and binding of an antibody to platelet factor IV is associated with heparin-induced thrombocytopenia.

A 71-year-old male patient with a history of myocardial infarction (MI) and peripheral vascular disease (PVD) has been advised by his family physician to begin taking 81 mg aspirin once daily. Which of the following statements best captures an aspect of the underlying rationale for the physician's suggestion? A) Platelet aggregation can be precluded through inhibition of prostaglandin production by aspirin. B) Aspirin helps to inhibit ADP action and minimizes platelet plug formation. C) Aspirin can reduce unwanted platelet adhesion by inhibiting TXA2 synthesis. D) Aspirin inhibits the conversion of fibrinogen into fibrin and consequent platelet plug formation.

A Feedback: Aspirin prevents platelet plug formation by inhibiting synthesis of prostaglandins that mediate clot formation. Aspirin does not influence ADP, TXA2 synthesis, or fibrinogen conversion.

In the context of a workshop on rheumatoid arthritis, a clinical educator is teaching a group of nurses about autoimmune diseases. Which of the following statements by an attendee would the educator most likely want to follow up with further teaching? A) "Introduction of a foreign antigen can sometimes induce a cascade of immune response that is not self-limiting" B) "Often the problem can be traced to antigens that sensitize T cells without the need for presentation." C) "In some cases, the body attacks its own cells that are chemically similar to those of infectious organisms." D) "Sometimes when the body's own cells are released after a long time, they are interpreted as being foreign."

A Feedback: Autoimmune responses are not considered to be uncontrolled cascades of immune response that are catalyzed by introduction of an antigen. Superantigens are able to forego the normal antigen presentation process and directly stimulate T-cell response, resulting in overactivation of T cells. Molecular mimicry involves the misidentification of somatic cells as similar foreign cells, and self-antigens that have been sequestered for long periods can invoke an immune response.

The nurse knows which of the following statements listed below is accurate regarding the functions and nature of cytokines relative to a variety of pathologies? A) "A particular cytokine can have varied effects on different systems, a fact that limits their therapeutic use." B) "Cytokine production is constant over time, but effects are noted when serum levels cross a particular threshold." C) "Most cytokines are produced by granular leukocytes, and different cells are capable of producing the same cytokine." D) "Cytokine actions are self-limiting, in that activation of one precludes activation of other cytokines with similar actions."

A Feedback: Because cytokines can mediate diverse effects due to their pleiotropic properties, they can have significant side effects. Cytokine production is brief, not constant, and production does not normally take place in granulocytes. Activation of a cytokine does not necessarily limit other similar cytokines, and additive effects are not uncommon.

All antiretroviral medications interfere with some stage of the HIV life cycle. What stage do protease inhibitors prevent? A) Cleavage of the polyprotein chain into the individual proteins that will be used to make new virus B) Addition of more nucleosides to the DNA chain C) Killing of the CD4+ T cell to release virions into the bloodstream D) Attachment of the virus to CD4+ cell receptors

A Feedback: By binding to the protease enzyme and inhibiting its function, protease inhibitors prevent cleavage of the polyprotein chain into individual proteins. Virions are still released into the body, but they are immature and noninfectious.

A patient diagnosed with low-risk chronic lymphocytic leukemia (CLL) has recently developed thrombocytopenia. One of the medications utilized to treat this would be A)dexamethasone, a corticosteroid. B)cisplatin, a chemotherapeutic. C)vincristine, a Vinca alkaloid. D)doxorubicin, a cytotoxic antibiotic.

A Feedback: Complications of CLL include thrombocytopenia and may require treatment with corticosteroids or splenectomy. None of the other medications are used for treatment of CLL.

The nurse will most likely assess which of the following clinical manifestations in a client who was diagnosed with Creutzfeldt-Jakob disease? A) Change in behavior and memory and loss of coordination leading to encephalopathy B) Gastrointestinal problems like vomiting and diarrhea C) Muscle inflammation and edema, making movements very painful D) Projectile vomiting, hypertension, and drowsiness caused by elevated ICP

A Feedback: Creutzfeldt-Jakob disease is collectively called transmissible neurodegenerative disease that is characterized by a slowly progressive, noninflammatory neuronal degeneration and leads to a loss of coordination, dementia, and death. With this disease, encephalopathy is the primary presenting factor. GI problems, muscle inflammation, and ICP are not clinical manifestations associated with this prion.

A 30-year-old woman who has given birth 12 hours prior is displaying signs and symptoms of disseminated intravascular coagulation (DIC). The client's husband is confused as to why a disease of coagulation can result in bleeding. Which of the nurse's following statements best characterizes DIC? A)"So much clotting takes place that there are no available clotting components left, and bleeding ensues." B)"Massive clotting causes irritation, friction, and bleeding in the small blood vessels." C)"Excessive activation of clotting causes an overload of vital organs, resulting in bleeding." D)"The same hormones and bacteria that cause clotting also cause bleeding."

A Feedback: DIC hemorrhage results from an insufficiency of clotting proteins after large-scale coagulation. It is not a result of physical irritation, organ overload, or bacteria and hormones.

Which of the following phenomena best accounts for the increased presence of leukocytes at the site of inflammation? A) Existing leukocytes stick to the epithelial cells and move along blood vessel walls. B) Increased numbers of leukocytes are released into circulation via cytokine stimulation. C) Leukocytes are osmotically drawn from circulation into the interstitial space as a result of swelling. D) Epithelium expresses leukocyte stimulation factors in response to cell injury.

A Feedback: During inflammation, leukocytes accumulate at the point of epithelial contact in the processes of margination, adhesion, and transmigration. This is not directly achieved by a way of increased leukocyte production or release, nor by osmotic pressure. The epithelium does not produce leukocyte stimulation factors.

A deficiency in which of the following would result in an inhibition of the inflammatory response? A)Histamine B)Helper T cells C)B cells D)Vitamin K

A Feedback: Histamine is a key mediator in the inflammatory system, unlike helper T cells, B cells, or vitamin K.

Of the following list of patients, who would likely benefit the most from hyperbaric oxygen therapy? A) A trauma patient who developed Clostridium spp., an anaerobic bacterial infection in his femur B) A patient who developed a fistula between her bowel and vagina following cervical cancer surgery C) A school-aged child who fell on gravel and has terrible road rash D) A football player who has torn a meniscus in his knee for the third time this year

A Feedback: Hyperbaric oxygen is a treatment that has demonstrated improved wound healing in multiple types of injuries. It enhances wound healing by a number of mechanisms, including the increased killing of bacteria by neutrophils, impaired growth of anaerobic bacteria, and the promotion of angiogenesis.

Following a course of measles, a 5-year-old girl developed scattered bruising over numerous body surfaces and was diagnosed with immune thrombocytopenic purpura (ITP). As part of her diagnostic workup, blood work was performed. Which of the following results is most likely to be considered unexpected by the health care team? A)Increased thrombopoietin levels B)Decreased platelet count C)Normal vitamin K levels D)Normal leukocyte levels

A Feedback: In ITP, thrombopoietin levels are not elevated. Platelet levels would be expected to be low, and vitamin K and leukocyte levels would be unlikely abnormal.

Which of the following individual situations listed below best exemplifies the processes of innate immunity? A) A child who has experienced heat and swelling of his skinned knuckle B) An adult who complains of itching and is sneezing because he is allergic to pollen C) A client whose blood work indicates increased antibody titers during an acute illness D) A client who has experienced rejection of a donor liver after transplantation

A Feedback: In a skinned knuckle, one of the body's main innate defenses, the skin, is breached. The heat and swelling that accompany a breach in the skin are inflammatory responses, part of the body's innate immune defenses. Allergies are an inappropriate adaptive response mediated by immunoglobulin E; antibody titers increase during illness in response to the infection; and transplanted organs are rejected because the organ is recognized as foreign. These are all aspects of specific, acquired immunity.

A nurse is providing care for a client who is immunocompromised following chemotherapy. The nurse knows which of the following characterizations of the adaptive immune system listed below is responsible for the client's disruption in his normal immune function? A) Epitopes on antigens are recognized by immunoglobulin receptors following presentation by accessory cells. B) Haptens combine to form epitopes that stimulate the response of regulatory and effector cells. C) Effector cells orchestrate the immune response of regulatory cells toward an antigen. D) Accessory cells such as macrophages are engulfed by regulatory cells, stimulating effector cells.

A Feedback: In the adaptive immune response, accessory cells present antigen epitopes to receptors, initiating the immune response of lymphocytes. Epitopes may combine to form haptens, and regulatory cells orchestrate effector cells. Regulatory cells do not engulf accessory cells.

A client with a diagnosis of atrophic gastritis and consequent pernicious anemia is receiving high oral doses of vitamin B12. Which of the following changes would be most likely expected by his care provider at the completion of his treatment? A)Decreased mean corpuscular volume B)Increased serum bilirubin C)Increased folic acid levels D)Decreased free heme levels

A Feedback: Increased red cell size is associated with vitamin B12 deficiency, and MCV would decrease with treatment. Increased bilirubin and folic acid levels would not be associated with resolution of pernicious anemia, and heme is not normally present or measured in a free circulatory form.

Which of the following patients would be most likely to be experiencing an increase in renal erythropoietin production? A)A 71-year-old smoker admitted to hospital with exacerbation of his chronic obstructive pulmonary disease (COPD) B)A 70-year-old woman admitted with dehydration secondary to an overdose of her potassium-wasting diuretic C)A 68-year-old man with a long-standing diagnosis of polycythemia vera D)A 21-year-old man with acute blood loss secondary to a motor vehicle accident 3 hours prior

A Feedback: Increases in erythropoietin production are associated with secondary polycythemia, and not polycythemia vera, a health problem that can be induced by the hypoxia resulting from smoking and lung disease. Dehydration is associated with relative polycythemia, and sudden blood loss would not manifest in increased erythropoietin production.

Which of the following statements by a client of a cancer center who has a new diagnosis of non-Hodgkin lymphoma (NHL) demonstrates a sound understanding of the diagnosis and treatment of the health problem? A)"They confirmed my diagnosis with a lymph node biopsy, and I'll get radiation treatment soon because it's fairly early stage." B)"They took a sample of my lymph nodes, and I'll be having surgery soon that will hopefully cure my lymphoma." C)"My blood work came back positive for NHL, and I'm meeting with my oncologist to discuss chemotherapy soon." D)"Since the tests show NHL, I'm going to pursue my options for palliative care because I'm committed to dying with dignity."

A Feedback: Lymph node biopsy is often used to provide a diagnosis of NHL, and radiation is a common treatment, especially in early stages of the disease. Surgery is not a noted treatment modality, and while blood work may help with staging, it is not a common source of a confirming diagnosis. Treatment options do exist, and palliative care would be premature.

A 10-year-old child with strep throat asks the nurse, "why there are large bumps [lymph nodes] on my neck when my throat gets sore?" The nurse replies lymph nodes A) help your body fight off infections by allowing special cells (lymphocytes and macrophages) move through the lymph chain and engulf and destroy germs. B) bring in cells into the lymph node (your bump) to stop the germs from going anywhere else in the body. C) bring all kind of good cells to your throat so that they can wall the strep off and keep the germs from getting any food or water." D) help your tonsils get bigger with cells that will bring immune cells into your throat to prevent any other infections.

A Feedback: Lymphocytes and macrophages move slowly through the lymph nodes so that they can have adequate time to engulf microorganisms and interact with circulating antigens. The lymph nodes do not bring cells into the node to stop germs; do not bring cells to the throat to wall off strep; and do not enlarge the tonsil with immune cells.

A 67-year-old patient diagnosed with myasthenia gravis will likely display which clinical manifestations as a result of autoantibodies ultimately blocking the action of acetylcholine, resulting in destruction of the receptors? A) Weakness of the eye muscles; difficulty in swallowing and slurred speech; impaired gait B) Tremor of hands/arms; rigidity of the arms; shuffling gait C) Short-term memory lapses; problems with orientation; a lack of drive or initiative D) Facial droop; slurred speech; weakness on one side of the body

A Feedback: Myasthenia gravis ultimately results in destruction of receptors in the neuromuscular junction leading to a decrease in neuromuscular function. Answer B relates to s/s of Parkinson disease; answer C relates to Huntington disease; and answer D relates to classic s/s of CVA.

A school-age child with a history of asthma has brought a note home from school stating that there has been one case of meningitis (Neisseria meningitides) in the school. Since the mother is a nurse, she is very concerned since she knows the portal of entry of this pathogen is A) by inhalation via the respiratory tract such as through breathing or yawning. B) by direct contact with a contaminated object such as a pencil. C) by ingestion such as when children share their drink with their friends. D) through a cut or abrasion that may occur on the playground.

A Feedback: Neisseria meningitides is one of a number of pathogens that invade the body through the respiratory tract.

Following an injury resulting in a small cut from a knife, the first cells to go to the area of the cut would be the A)neutrophils. B)erythrocytes. C)albumin. D)basophils.

A Feedback: Neutrophils are very mobile and are the first cells to go to an area of tissue damage. Erythrocytes (RBCs) contain an O2-carrying protein that functions in the transport of oxygen. Albumin, a plasma protein, does not pass through the pores in the capillary wall to enter the interstitial fluid and therefore contributes to the plasma osmotic pressure and maintenance of blood volume. Basophil granules contain heparin, an anticoagulant; histamine, a vasodilator; and other mediators of inflammation.

A school nurse is teaching high school students about HIV and AIDS in the context of the school's sexual health curriculum. Which of the students' following statements would the nurse most likely want to correct or clarify? A) "They have to take a blood sample from you in order to test you for AIDS." B) "Drugs for AIDS reduce the virus in your body, but they don't get rid of it." C) "Lots more heterosexual people get HIV these days than they used to." D) "Condoms provide really good protection from AIDS."

A Feedback: Oral tests now exist for preliminary diagnosis of HIV. Medications for AIDS do not cure the disease, and incidence is increasing among heterosexuals. Condoms provide effective protection from the virus.

After several months on a waiting list, a 44-year-old male received a liver transplant 5 days ago. In the last 36 hours, he has developed a rash beginning on his palms and soles, along with abdominal pain and nausea. It has been determined by his care team that the immune response that is causing his symptoms originates not with his own compromised immune components but with those introduced with his new organ. This man's most likely medical diagnosis is A)graft versus host disease (GVHD). B)acute transplant rejection. C)hyperacute organ rejection. D)T-cell-mediated graft rejection.

A Feedback: Rash, gastrointestinal involvement, and pernicious activity by donor immune cells are the hallmarks of GVHD. The description does not suggest acute or hyperacute transplant rejection, and T-cell-mediated rejection is not a diagnosis in and of itself, but rather one of the mechanisms of transplant rejection.

A nurse is changing the wound dressing on the coccyx-region pressure ulcer of an immobilized patient. The existing dressing is saturated with both watery, clear discharge and foul, gray-colored liquid. Which of the following entries in the patient's chart best captures this? A)"Large amounts of suppurative and serous exudates noted" B)"Purulent discharge and fibrinous exudates noted on existing dressing" C)"Abscess activity noted to coccyx wound" D)"Plasma proteins and membranous exudates present on existing dressing"

A Feedback: Serous discharge is clear and low in plasma proteins, while suppurative, or purulent, exudates are a mass of degraded cells. An abscess would be physically contained with no discharge, and the exudate is neither fibrinous nor membranous.

Which of the following colony-stimulating factors (CSFs) is given to ESRD patients to help with their chronic anemia? A)Erythropoietin (EPO) B)Thrombopoietin (TPO) C)Neupogen D)Interleukin

A Feedback: The CSFs that act on committed progenitor cells include erythropoietin (EPO), which stimulates RBC production. TPO stimulates platelets. Neupogen is for white blood cell production. Interleukin is a cytokine utilized as an immune response agent.

A hospital laboratory technician is performing routine blood analysis as part of an inpatient's assessment and is examining the sample in a test tube following processing in a centrifuge and the addition of an anticoagulant. Which of the following observations would the technician most likely interpret as an anomaly? A) The bottom layer of blood in the tube accounts for around one third of the total volume. B) The middle layer of cells appears white to gray in color. C) The top layer of cells is too thin to visualize without microscopy. D) The yellowish fluid on the top of the sample appears to constitute around one half of the total volume.

A Feedback: The body layer of cells in a centrifuged blood sample consists of erythrocytes and should constitute around 42% to 47% of the total blood volume. The middle cell layer of leukocytes should appear whitish gray, and the thrombocytes above that layer are frequently not discernable. The plasma layer should account for around 55% of the total blood volume.

A 40-year-old male who has been HIV positive for 6 years is experiencing a new increase in his viral load along with a corresponding decrease in his CD4+ count. Which of the following aspects of his immune system is likely to remain most intact? A) Presentation of major histocompatibility molecules on body cells B) Orchestration of natural killer cells as part of cell-mediated immunity C) Activation of B lymphocytes D) Phagocytic function of monocytes and macrophages

A Feedback: The expression of MHC on various cells of the body is not noted to be directly influenced by HIV. However, infected CD4+ cells are compromised in their ability to guide the action of NK cells, to direct phagocytic function of macrophages, and to present antigens that activate B cells.

Which of the following glycoproteins is responsible for treating such diseases as bone marrow failure following chemotherapy and hematopoietic neoplasms such as leukemia? A)Growth factors and cytokines B)Neutrophils and eosinophils C)T lymphocytes and natural killer cells D)Natural killer cells and granulocytes

A Feedback: The identification and characterization of the various growth factors and cytokines have led to their use in treating a wide range of diseases like bone marrow failure, hematopoietic neoplasms, infectious diseases, and congenital and myeloproliferative disorders. Neutrophils are primarily responsible for maintaining normal host defenses against invading bacteria, cell debris, or foreign substances. Eosinophils increase in number during allergic reactions and parasitic infections. T lymphocytes are involved in cell-mediated immunity. Natural killer cells participate in innate or natural immunity, and their function is to destroy foreign cells. Granulocytes are phagocytic cells.

The nurse knows which of the following statements listed below best matches the phase of the infectious process of food poisoning with a client with sudden, violent diarrhea and vomiting after consuming chicken and potato salad 8 hours ago at the beach on a hot day? A)Maximum impact of infectious process B)Insidious prodromal phase C)Sudden incubation of active replication D)Subacute prodromal phase

A Feedback: The lack of prodrome and intense symptom onset typify a fulminant illness. The onset described does not characterize a prodromal phase or incubation.

When explaining the final stages of the inflammatory response to pathogens, the nurse will educate the patient about A) how the body can kill the pathogen by generating toxic oxygen and nitrogen products producing such things as nitric oxide and hydrogen peroxide. B) margination, which is a process whereby white cells (leukocytes) stick to the endothelium and accumulate along the blood vessel. C) the increase in vascular permeability, which lets fluids leak into the extravascular tissues. D) the promotion of tissue regeneration whereby monocytes and macrophages produce potent prostaglandins and leukotrienes.

A Feedback: The latter stages of phagocytosis results in intracellular killing of pathogens accomplished by several mechanisms, including toxic oxygen and nitrogen products, lysozymes, proteases, and defensins. The metabolic burst pathways generate toxic oxygen and nitrogen products (i.e., nitric oxide, hydrogen peroxide, and hypochlorous acid). Margination is one of the early stages of the inflammatory response. Vascular changes occur with inflammation but are prior to the final stage. Macrophages arrive within hours at the inflammation site.

In which of the following individuals would a clinician most suspect multiple myeloma as a diagnosis? A)A 40-year-old man who has had three broken bones over the past 6 months and whose serum calcium and creatinine levels are elevated B)A 68-year-old former coal miner who has white cell levels exponentially higher than normal ranges C)An 81-year-old male resident of a long-term care home who has an uncommon bacterial pneumonia and who is unable to produce a fever D)A 70-year-old woman whose blood work reveals large numbers of immature granulocytes

A Feedback: The main sites involved in multiple myeloma are the bones and bone marrow. In addition to the abnormal proliferation of marrow plasma cells, there is proliferation and activation of osteoclasts that lead to bone resorption and destruction. This increased bone resorption predisposes the individual to pathologic fractures and hypercalcemia. Many patients also present with renal insufficiency. Leukostasis, susceptibility to infection, and disorders of granulocyte development are not hallmarks of multiple myeloma.

During science class, a student asks, "What's the difference between plasma and serum in the blood?" The nurse responds that the primary difference between plasma and serum is that plasma contains A)fibrinogen. B)heparin. C)white blood cells. D)hydrogen ions.

A Feedback: The major difference between plasma and serum is the presence of fibrinogen in the plasma from an anticoagulated centrifuged whole-blood specimen. When blood is removed from the body for testing, it clots within 30 to 60 minutes. The clot contains the blood's cellular components enmeshed in an insoluble fibrin network (formed by the polymerization of soluble plasma protein fibrinogen). The remaining fluid portion is the yellow liquid serum. This serum no longer contains fibrinogen because the fibrinogen originally present in uncoagulated blood was used up in the formation of the blood clot. Heparin, WBC, and hydrogen ions are incorrect.

Following a bone marrow biopsy, which of the following assessments would indicate the patient is experiencing a complication as a result of this diagnostic procedure? A) BP 90/60, heart rate 132, excess bleeding, and hematoma noted at the insertion site B) BP 130/80, oxygen saturation 95%, and crackles heard on inspiration C) Respiratory rate 24 and complaining of pain at insertion site D) Heart rate regular 64 beats/minute and temperature 99.6°F orally

A Feedback: The major hazard of a bone marrow biopsy is the risk of hemorrhage. This is usually seen by a decrease in BP, tachycardia, edema from blood pooling in the tissues, etc. All of the other answers are s/s other than shock. Crackles in the lungs correlate with fluid/secretions in the lungs. A bone marrow aspiration is usually performed on the posterior iliac crest and should not cause an elevated respiratory rate. A low-grade fever is not considered a complication.

A 2-year-old girl has had repeated ear and upper respiratory infections since she was born. A pediatrician has determined a diagnosis of transient hypogammaglobulinemia of infancy. What is the physiological origin of the child's recurrent infections? A) The child's immune system is unable to synthesize adequate immunoglobulin on its own. B) The child had a congenital absence of IgG antibodies that her body is only slowly beginning to produce independently. C) The child was born with IgA and IgM antibodies, suggesting intrauterine infection. D) The child lacks the antigen-presenting cells integral to normal B-cell antibody production.

A Feedback: Transient hypogammaglobulinemia of infancy is characterized by inadequate communication between B and T cells. IgG would be inherited through placental blood, and an intrauterine infection is neither causative or nor synonymous with transient hypogammaglobulinemia of infancy. The diagnosis does not include a lack of antigen-presenting cells.

A 32-year-old woman presents at her neighborhood health clinic complaining of weakness and a feeling of abdominal fullness. She reports that 6 months earlier she noticed that she had difficulty in maintaining the high level of energy she has relied on during her aerobic workouts over the past few years. Because she felt that she was in overall good health, but knew that women often need additional iron, she added a multiple vitamin with iron and some meat and leafy greens to her diet. She followed her plan carefully but had no increase in energy. Upon examination, her spleen is noted to be enlarged. Which of the following is most likely to be the cause? A)Accelerated CML B)Stage A Hodgkin disease C)Infectious mononucleosis D)CLL

A Feedback: Typically, CML follows a triphasic course: (1) a chronic phase of variable length, (2) a short accelerated phase, and (3) a terminal blast crisis phase. The onset of the chronic phase is usually slow with nonspecific symptoms. Anemia causes weakness, easy fatigability, and exertional dyspnea. The accelerated phase is characterized by enlargement of the spleen and progressive symptoms. Persons with Hodgkin disease are commonly designated as stage A if they lack constitutional symptoms and stage B if significant weight loss, fevers, pruritus, or night sweats are present. In cases of infectious mononucleosis, most persons seek medical attention for severe pharyngitis, which usually is most severe on days 5 through 7 and persists for a total of 7 to 14 days. CLL is mainly a disorder of older persons; fewer than 10% of those who develop the disease are younger than 50 years of age. Men are affected twice as often as women.

Which of the following statements most accurately conveys an aspect of lymphatic system activity? A)B- and T-lymphocyte development begins in the bone marrow and ends in the peripheral lymphoid structures. B)B cells and macrophages are released from the bone marrow in their completed state. C)Stem cells in the lymph nodes initiate and regulate the process of white cell synthesis. D)Leukocytes bypass vascular circulation and are distributed instead by the lymphatic system.

A Feedback: While both precursor B and T lymphocytes begin their development in the bone marrow, they migrate to peripheral lymphoid structures to complete the differentiation process. Stem cells are not located in the lymph nodes, and circulation of white cells is not exclusive to the lymphatic circulatory system.

A patient with rheumatoid arthritis has been diagnosed with a secondary immune-associated neutropenia called Felty syndrome. The nurse has had to research this and found that she should be assessing this patient for which of the following manifestations of Felty syndrome. Select all that apply. A)Upper left quadrant pain on palpation B)An area of diminished breath sounds related to pneumonia C)Intermittent pain that radiates from the flank to the groin D)Swelling and pain in all joints when put through full range of motion E)Headache that worsens when exposed to bright lights

A, B Feedback: Felty syndrome, a variant of rheumatoid arthritis (RA), is a triad of splenomegaly (upper left quad pain), recurrent pulmonary infections (diminished breath sounds), and neutropenia. Intermittent pain that radiates from the flank to the groin is usually associated with kidney stone pain. Swelling and pain in all joints when put through full range of motion are also associated with rheumatoid arthritis.

A patient diagnosed with a primary immunodeficiency disorder has asked his siblings to be tested as possible stem cell donors. When discussing this procedure with his family, the nurse emphasizes that stem cells can be harvested from: Select all that apply. A)bone marrow. B)peripheral blood. C)skin tissue harvesting. D)mouth swabs. E)tears.

A, B Feedback: Hematopoietic stem cells can be harvested from either the bone marrow or the peripheral blood and also from the umbilical cord blood. From sibling donors, the results are effective with improved survival in approximately 90% of people.

A patient comes into a clinic complaining of cough, fever, and shortness of breath. The patient informs the health care provider that he is HIV positive. Upon physical exam, the family nurse practitioner (FNP) may note which of the following clinical manifestations of suspected Pneumocystis jiroveci pneumonia (PCP)? Select all that apply. A) Interstitial infiltrates on chest x-ray B) Respiratory rate of 32 with normal breath sounds C) Stridor when taking a deep breath D) Use of abdominal muscles to breathe while sitting on the exam table E) Night sweats that require clothing changes frequently throughout the night

A, B Feedback: PCP is a common presenting manifestation of AIDS or people with compromised immune systems. The symptoms include cough, fever, shortness of breath, and weight loss. Physical exam demonstrates only fever and tachypnea (elevated respiratory rate) and normal breath sounds. Chest x-ray shows interstitial infiltrates. Night sweats are usually associated with tuberculosis infection.

A newly diagnosed HIV-positive adolescent has blood work drawn, which includes a CD8 T-cell count. The nurse knows which of the following functions of CD8 T cells listed below will assist the adolescent's immune system in fighting off the viral attack? Select all that apply. A) Release destructive enzymes B) Trigger intracellular programmed death C) Cause allergens to surround the virus D) Boost antigen-antibody response E) Remove foreign material from lymph before it enters the blood

A, B Feedback: The primary function of cytotoxic T (CD8) cells is to monitor the activity of all cells in the body and destroy any that threaten the integrity of the body. The CD8 cells destroy target cells by releasing cytolytic enzymes, toxic cytokines, and pore-forming molecules or by triggering membrane molecules and intercellular apoptosis. Removal of foreign material from lymph before it enters the blood is the function of lymph nodes.

A newly diagnosed leukemia patient begins hemorrhaging from every orifice. The physician is concerned that the patient has developed disseminated intravascular coagulation (DIC). The nurse should anticipate which of the following orders to be prescribed for this patient? Select all that apply. A)Transfuse 2 units of platelets. B)Transfuse fresh frozen plasma. C)Give aspirin twice per day. D)Administer IV Toradol stat. E)Place in reverse isolation.

A, B Feedback: The treatment of DIC is directed toward managing the primary disease, replacing clotting components, and preventing further activation of clotting mechanisms. Transfusions of FFP, platelets, or fibrinogen-containing cryoprecipitate may correct the clotting factor deficiency. ASA would make the bleeding worse. Toradol is an NSAID and should be avoided in patients with a bleeding problem. Reverse isolation is implemented for patients with pancytopenia.

Two years after chemotherapy and radiation therapy for lung cancer, a 72-year-old patient notices that he seems to be extremely tired all the time. The physician suspects the patient may have developed aplastic anemia. The nurse assessing the patient will likely find which of the following clinical manifestations of aplastic anemia? Select all that apply. A)Complaints of weakness and fatigue B)Small spots of skin hemorrhages over the entire body C)Excess bleeding from gums and nose D)Spoon-shaped deformity of the fingernails E)Hemolysis from renal dialysis treatments.

A, B, C Feedback: The onset of aplastic anemia may be insidious or sudden. The initial presenting symptoms include weakness, fatigue, and pallor caused by the anemia. Petechiae and ecchymoses often occur on the skin, and bleeding from the nose, gums, vagina, or GI tract may occur due to decreased platelet levels. Spoon-shaped deformity of the fingernails is seen in iron deficiency anemia. Hemolysis and blood loss from renal dialysis treatments contribute to anemia associated with a deficiency of erythropoietin (which is normally produced in the kidneys).

Which of the following signs and diagnostic findings are recognized components of the metabolic and morphologic changes that occur with HIV infection accompanied with lipodystrophy? Select all that apply. A)Hyperlipidemia B)Insulin resistance C)Deficiencies of anterior pituitary hormones D)increased abdominal girth E)Breast enlargement

A, B, D, E Feedback: Hyperlipidemia and insulin resistance are aspects of lipodystrophy, a phenomenon that also frequently includes breast enlargement and increased abdominal girth. Pituitary hormone deficiencies are not a noted component of HIV-related metabolic changes.

In the ICU, a patient has been diagnosed with sepsis due to a bacterial invasion. The human body usually responds to infections by developing an uncontrolled inflammatory response with large production and release of inflammatory cytokines such as IL-1 and TNF-a. The nurse will note which of the following clinical manifestations in this septic patient as a result of the activation of these cytokines? Select all that apply. A)Excessive interstitial edema related to increased vascular permeability B)Decreased cardiac output resulting from myocardial depression C)Increased respiratory rate with crackles heard throughout all lung fields D)Excessive bleeding from bowels and bladder E)Lower blood pressure due to intravascular fluid loss

A, B, E Feedback: SIRS (systemic inflammatory response syndrome) can develop as a result of large quantities of microbes entering the blood, resulting in the release of enormous quantities of inflammatory cytokines. They cause generalized vasodilation (hypotension), increased vascular permeability (fluid loss into the tissues), intravascular fluid loss (dehydration with low urine output and low BP), myocardial depression (decreased cardiac output), and circulatory shock.

Which of the following nursing interventions would be a priority when caring for a newborn who is receiving phototherapy for high bilirubin levels? Select all that apply. A)Frequent monitoring of temperature B)Keeping diapers dry and clean C)Maintaining oral intake to prevent dehydration D)Putting lotion on his skin frequently to prevent drying/cracking of skin E)Applying sunscreen to prevent ultraviolet radiation

A, C Feedback: Effective treatment depends on the area of skin exposed and the infant's ability to metabolize and excrete bilirubin. Frequent monitoring of bilirubin levels, body temperature, and hydration is critical to the infant's care. Diapers are usually not worn when under phototherapy light (want to keep the skin exposed to the light). Applying lotion and sunscreen would result in possible burning of the infant's skin while under the light.

Which of the following patients would be at risk for developing nonthrombocytopenic purpura? Select all that apply. A) A child adopted from India and displaying malaise, lethargy, and petechiae all over the body B) A 73-year-old patient admitted with concussion that resulted from a fall C) A 55-year-old patient diagnosed with Cushing disease displaying bruises, weight gain with a buffalo hump, and moon face D) A 15-year-old insulin-dependent diabetic with hypoglycemia displaying irritability with headaches and tachycardia E) A pregnant mother experiencing headaches and proteinuria

A, C Feedback: Vascular disorders that cause bleeding include vitamin C deficiency (answer A), Cushing disease (answer C), senile purpura, and aging process. Answers B and D do not result in weak vessel walls initially. Pregnant mother experiencing headaches and proteinuria is associated with preeclampsia.

Tumor necrosis factor-a and IL-1 are major cytokines that mediate inflammation. If the patient is developing a systemic response to an infection, the nurse will likely assess which of the following clinical manifestations? Select all that apply. A)Elevated temperature B)Hypertension C)Tachycardia D)Decrease in urine output E)Anorexia

A, C, E Feedback: IL-1 and TNF-a are mediators of the acute-phase responses associated with infection or injury. Features of these systemic responses include fever (elevated temperature), hypotension, tachycardia (increased heart rate), anorexia, increase in neutrophil count, and increased levels of corticosteroid hormones.

A 13-year-old African American boy comes to the ER complaining of fatigue and a rapid heartbeat. In conversation with the father, it becomes apparent to you that the boy has grown 2 inches in the previous 5 months. What is the first problem the health care team would attempt to rule out? A)Sickle cell anemia B)Iron deficiency anemia C)Thalassemia D)Aplastic anemia

B Feedback: Although each of the above answers is associated with fatigue and rapid heartbeat, male adolescents are particularly susceptible to iron deficiency anemia. They have high iron requirements because of growth spurts and dietary deficiencies.

A male elementary school student has a severe allergy to peanuts and is displaying the signs of anaphylactic shock after inadvertently eating a peanut-containing candy bar. Which of the following statements best captures the boy's current status and preferred treatment? A) He is experiencing shortness of breath caused by potent vasoconstriction that can be relieved by epinephrine injection. B) He is approaching vascular shock and developing edema due to actions of IgE antibodies, situations that can be reversed by administration of epinephrine. C) His mast cells and basophils have been sensitized, but systemic effects can be mitigated by administration of bronchodilators. D) He is likely in a primary- or initial-stage allergic response that can be relieved by antihistamine administration.

B Feedback: Anaphylactic reactions are often accompanied by vascular shock and edema, and the normal treatment is epinephrine injection. Symptoms are caused by vasodilation, not vasoconstriction; bronchodilators are not the ideal treatment.

A 14-year-old boy has been diagnosed with infectious mononucleosis. Which of the following pathophysiological phenomena is most responsible for his symptoms? A)The Epstein-Barr virus (EBV) is lysing many of the boy's neutrophils. B)Viruses are killing some of his B cells and becoming incorporated into the genome of others. C)The EBV inhibits the maturation of white cells within his peripheral lymph nodes. D)The virus responsible for mononucleosis inhibits the maturation of myeloblasts into promyelocytes.

B Feedback: B lymphocytes all of which have receptors for the EBV that causes mononucleosis. Infection of the B cells may take one of two forms—it may kill the infected B cell, or it may become incorporated into its genome. Lysis of neutrophils or inhibition of white cell maturation or differentiation is not a central component of mononucleosis etiology.

Which of the following aspects of a patient's site of inflammation would help the care provider rule out chronic inflammation? A)High levels of macrophages B)Increased neutrophils C)Proliferation of fibroblasts D)Infiltration of lymphocytes

B Feedback: Chronic inflammation lacks the sudden and marked proliferation of neutrophils that is associated with acute inflammation. Chronic inflammation is associated with increased presence and action of fibroblasts, macrophages, and lymphocytes.

A patient asks the health care provider why his lower legs look purple. The health care provider will base her response on which pathophysiological principle? A) Too much trauma breaks capillaries, and they bleed into the tissue. B) The bruising around the ankles is due to the fact that it is a dependent area where the capillary pressure is higher. C) There is a problem with his plasminogen levels. D) Morbid obesity causes veins to enlarge and bleed into tissues due to stress the abdomen is placing on the vascular system.

B Feedback: Cutaneous bleeding is seen as pinpoint hemorrhages and purple areas of bruising in dependent areas where the capillary pressure is higher. There is no indication that the patient has experienced trauma to the area or is morbidly obese. Plasminogen helps with clot dissolution.

Which of the following patients is most likely to benefit from transplantation of thymic tissue or major histocompatibility complex (MHC)-compatible bone marrow? A) A 12-year-old girl with a history of epilepsy and low IgG levels secondary to phenytoin use B) A 7-year-old boy whose blood work indicates decreased IgA and IgG with increased IgM C) A 6-year-old boy whose pre-B cells are incapable of translation to normal B cells D) A 9-year-old girl who has a diagnosis of IgA deficiency

B Feedback: Decreased IgA, IgE, and IgG with increased IgM levels are characteristic of X-linked immunodeficiency with hyper-IgM, a primary cell-mediated immunodeficiency that would respond to thymic tissue transplantation and MHC-compatible bone marrow. Low IgG levels secondary to phenytoin use, X-linked hypogammaglobulinemia, and selective IgA deficiency are unlikely to be treated with the T-cell-focused treatments like thymic tissue transplantation and MHC-compatible bone marrow.

Which of the following statements best captures an aspect of the process of hematopoiesis? A) Colony-stimulating factors (CSFs) produce cytokines that activate progenitor cells. B) Progenitor cells differentiate into precursor cells. C) Various subtypes of pluripotent stem cells eventually differentiate into the cellular components of blood. D) Self-replicating precursor cells differentiate into specific CSFs.

B Feedback: During hematopoiesis, progenitor cells differentiate into precursor cells, which in turn differentiate into the cellular components of blood. Cytokine stimulation precedes and regulates the process, and there are no different subtypes of pluripotent stem cells. Precursor cells are not self-replicating.

Which of the following statements best conveys a characteristic of red blood cells? A)They lack organelles and soluble enzymes. B)They contribute to the maintenance of blood pH. C)They help maintain the body's fluid balance. D)They are self-replicating.

B Feedback: Erythrocytes contribute to acid-base balance through the production of bicarbonate and hydrogen ions. They contain soluble enzymes but are neither self-replicating nor involved in fluid balance.

A family consumed some undercooked hamburger at a picnic and has since developed bloody diarrhea. The nurse knows which of the following statements listed below is correct regarding the infectious process? A) Evasive factors that become more virulent by evading parts of the host's immune system B) Exotoxins that damage vascular endothelium causing bleeding and low platelet counts C) Adhesion factors that can anchor a pathogen firmly to the host tissue surfaces D) Invasive factors capable of destroying the cell membrane by utilization of enzymes

B Feedback: Exotoxins can allow organisms to produce hemorrhagic colitis, which can be fatal. It is characterized by vascular endothelial damage, acute RF, and thrombocytopenia (low platelet counts). Toxic cell walls are classified as endotoxins. The ability to survive immune responses characterizes evasive factors. Adhesion factors and invasive factors are not involved in this situation.

A 44-year-old female patient presents to the emergency department with abnormal bleeding and abdominal pain that is later attributed to gallbladder disease. Which of the following diagnoses would the medical team be most justified in suspecting as a cause of the patient's bleeding? A)Excess calcium B)Vitamin K deficiency C)Hemophilia B D)Idiopathic immune thrombocytopenic purpura (ITP)

B Feedback: Factors VII, IX, and X and prothrombin require the presence of vitamin K for normal activity. Vitamin K deficiency may result from impaired fat absorption caused by liver or gallbladder disease. Calcium, factors X and V, and platelet phospholipids combine to form prothrombin activator, which then converts prothrombin to thrombin. Excess calcium could result in increased formation of blood clots. Hemophilia B is a hereditary disorder. Half of the cases of ITP occur as an acute disorder in children; ITP in adults is a chronic disorder with insidious onset.

A surgeon is explaining to the parents of a 6-year-old boy the rationale for the suggestion of removing the boy's spleen. Which of the following teaching points would be most accurate? A) "Ferritin is the activated and usable form of iron that your red blood cells can use to transport oxygen." B) "Ferritin is a stored form of iron that indirectly shows me whether you would benefit from iron pills." C) "Ferritin is a protein-iron complex that allows your red blood cells to make use of the iron that you consume in your diet." D) "Ferritin is the form of iron that is transported in your blood plasma to red blood cells that need it."

B Feedback: Ferritin is the protein-iron complex that is stored in tissues, especially the liver, and the serum levels can be used as an indicator of the need for iron supplements. It is not accurately characterized as an activated form of iron, nor does it mediate the actual use of iron by erythrocytes. Transferrin, not ferritin, is transported in plasma, and it is associated with iron storage.

A 24-year-old woman presents with fever and painful, swollen cervical lymph nodes. Her blood work indicates neutrophilia with a shift to the left. She most likely has A)a mild parasitic infection. B)a severe bacterial infection. C)a mild viral infection. D)a severe fungal infection.

B Feedback: Fever and painful, palpable lymph nodes are nonspecific inflammatory conditions; leukocytosis is also common but is a particular hallmark of bacterial infection. Neutrophilia also indicates a bacterial infection, whereas increased levels of other leukocytes would indicate other etiologies. The shift to the left--the presence of many immature neutrophils--indicates that the infection is severe, because the demand for neutrophils exceeds the supply of mature cells.

A nurse has just learned that her child has a life-threatening complement disorder known as hereditary angioneurotic edema (HAE). Due to deficiency in C1-INH, the nurse needs to be prepared for which possible life-threatening clinical manifestation? A)Bulging eyeballs B)Swelling of the airway C)Compressed carotid arteries D)Compression of brachial nerves

B Feedback: HAE is a rare, life-threatening complement disorder that results from deficiency of C1-inhibitor (HAE-C1-INH). It is an inherited autosomal dominant trait that causes mutation in the 11th chromosome. Deficiencies in C1-INH result in uncontrolled release of vasoactive substances that promote vascular permeability. The net result is the development of swelling in the subcutaneous tissues of the extremities, face/torso, or upper airway and GI tract. Laryngeal edema is a life-threatening manifestation that can lead to complete airway obstruction and death without interventions.

While teaching about HIV/AIDS to a group of high school seniors, the school health nurse will begin by explaining the basic facts that will likely include which of the following information? A) Like all viruses, HIV is a genetic material made from DNA with long molecules that carry genetic information. B) HIV is different from other viruses since it is a retrovirus that selectively attacks the body's immune cells. C) There are two types of HIV, but the one that is endemic to the United States is HIV type 2. D) HIV type 1 for some reason rarely develops into full-blown AIDS.

B Feedback: HIV is a retrovirus that selectively attacks the CD4+ T lymphocytes, the immune cells responsible for orchestrating and coordinating the immune response to infection. It must change from RNA to DNA through a series of stages in order to get in a cell and begin replication. HIV type 2 is endemic in West Africa but is rarely seen in other parts of the world. People with HIV-2 tend not to develop AIDS.

A student states, "It seems like helper T cells do a lot more than just 'help' the cellular immunity process." Which of the following responses listed below best conveys an aspect of the role of CD4+ helper T cells in immunity? A) "Without helper T cells, no antigens would be presented." B) "Helper T cells play a major role in stimulating and regulating the whole process." C) "Without helper T cells, the wrong antibodies would end up being produced." D) "Helper T cells are key to the hematopoiesis that produces all the components of the immune system."

B Feedback: Helper T cells are central to the regulation, proliferation, and stimulation of the immune system. They do not play a central role in antigen presentation or early hematopoiesis, however. Their absence would not result in incorrect antibody production, but rather insufficient or absent immune response.

A nurse is providing care for several patients on an acute medical unit of a hospital. Which of the following patients would be most likely to benefit from hematopoietic growth factors? A)A 51-year-old female patient with liver failure secondary to hepatitis B)A 61-year-old female patient with end-stage renal cancer C)A 55-year-old obese male patient with peripheral neuropathy secondary to diabetes D)A 44-year-old man with a newly diagnosed brain tumor

B Feedback: Hematopoietic growth factors are commonly used in cases of kidney disease and cancer. Erythropoietin (EPO) is a colony-stimulating factor that stimulates red blood cell production. As a review, erythropoietin is an essential hormone for red cell production. Without it, definitive erythropoiesis does not take place. Under hypoxic conditions, the kidney will produce and secrete erythropoietin to increase the production of red blood cells. With cancer of the kidney, the kidney is unable to perform its normal function. They are not as clearly indicated in cases of liver disease, diabetes, or brain tumors.

The nurse knows which of the following statements listed below relative to a client with malignant melanoma treated with alpha interferon (IFN-a) is accurate? Alpha interferon (IFN-a) A) will kill certain microorganisms that may help spread the cancer. B) plays an important role in the modulation of the inflammatory response. C) helps keep all the blood levels at a higher level. D) controls the migration of leukocytes to their primary site.

B Feedback: IFNs are cytokines that primarily protect the host against viral infections and play a role in the modulation of the inflammatory response.

When counseling a male patient with suspected HIV, the nurse informs him that if the enzyme-linked immunosorbent assay (ELISA) comes back positive, then A) no further testing is required since this confirms HIV infection. B) a second test known as the Western blot assay will be ordered to confirm positive HIV status. C) he will be sent to an infectious disease physician for a tissue biopsy to confirm infection. D) if the second test, the Western blot, returns negative, he has not developed a case of full-blown AIDS.

B Feedback: If ELISA is positive, his blood sample is then sent for Western blot assay. If the Western blot is positive, diagnosis of HIV is confirmed. If the Western blot is negative, then the person is not infected with HIV.

The family members of an elderly patient are wondering why his "blood counts" are not rising after his last GI bleed. They state, "He has always bounced back after one of these episodes, but this time it isn't happening. Do you know why?" The nurse will respond based on which of the following pathophysiological principles? A)"Everything slows down when you get older. You just have to wait and see what happens." B)"Due to stress, the red blood cells of older adults are not replaced as promptly as younger people." C)"The doctor may start looking for another cause of his anemia, maybe cancer of the bone." D)"Don't worry about it. We can always give him more blood."

B Feedback: In older adults, the number of progenitor cells declines. During a stress situation such as bleeding, the red blood cells of older adults are not replaced as promptly as those of their younger counterparts. Given the scenario, the patient is obviously bleeding from the GI tract. There is no reason to suspect the patient has bone cancer. Answer choice D is a nontherapeutic communication technique. The nurse is trying to pacify the family and not really addressing their concern.

Which of the following patients would be considered to be in the latent period of HIV infection? A) A 16-year-old prostitute who has open sores on her labia that drain purulent secretions B) A 33-year-old heroin drug abuser who has numerous enlarged lymph nodes in his axilla and cervical neck region for the past 4 months C) A 45-year-old alcohol abuser who is complaining of excessive vomiting of blood that started 2 weeks ago D) A 24-year-old college student who has developed a chronic cough that will not go away, even after taking two courses of antibiotics.

B Feedback: In the latent period, which can last up to 10 years, the CD4+ count falls gradually to approximately 200 cells/µL. Some people experience swollen lymph nodes that are chronically swollen for more than 3 months in at least two locations, not including the groin. The lymph nodes may be sore or visible externally.

Three days ago, a mother delivered her full-term infant who had been identified as having an in utero infection. The infant is receiving antibiotic and phototherapy, and the mother is breast-feeding. Which of the following types of immunoglobulins could most reasonably be expected to predominate in the infant's immune system? A)IgA, IgM, IgD B)IgG, IgA, IgM C)IgE, IgG, IgD D)IgM, IgD, Igm

B Feedback: Infants are born with IgG from transfer across the placenta, while IgA is found in colostrum. IgM is indicative of an in utero infection.

A female dental assistant has developed signs and symptoms of a latex sensitivity and is undergoing allergy testing as well as blood work. Which of the following components of the assistant's blood work would most likely be the focus of her health care provider's analysis? A)Analysis of class II MHC antigens B)Serum IgE immunoassays C)Serum B-lymphocyte levels D)Serum CD8+ levels

B Feedback: Latex sensitivity can be either a type I or type IV reaction. Though Th1 levels are relevant in a type IV reaction, IgE analysis is the most common component of relevant blood work. MHC and CD8+ levels are unlikely to be considered.

The nurse knows which of the following components listed below is needed for long-lasting immunity in a client with a diagnosis of sepsis without the causative agent identified? A)Neutrophils B)Lymphocytes C)Colony-stimulating factors D)Natural killer cells

B Feedback: Lymphocytes provide lifelong immunity and an antigen-specific response to harmful microorganisms. Neutrophils, macrophages, and natural killer cells do not provide this.

Which of the following patients has an absolute neutrophil count that is critically low and that the standard of care would recommend they be placed on neutropenic precautions? A)A patient on long-term steroids for rheumatoid arthritis with WBC of 7000 B)A 37-year-old patient with leukemia being treated with chemotherapy with ANC of 400 C)A 65-year-old prostate cancer patient receiving radiation therapy with neutrophil count of 2000 D)A 75-year-old renal failure patient receiving Epogen for anemia with hemoglobin level of 9.7.

B Feedback: Neutrophils constitute the majority of blood leukocytes and play a critical role in host defense mechanisms against infections. The ANC is supposed to be 1000/µL, and if the ANC is less than 500 cells/mm3, the person is generally put on neutropenic precautions in the hospital to protect him or her from the environment. A patient on long-term steroids for rheumatoid arthritis with WBC of 7000 has a normal WBC count; a prostate cancer patient receiving radiation therapy with neutrophil count of 2000 has a normal neutrophil count; a renal failure patient receiving Epogen for anemia with hemoglobin level of 9.7 is associated with RBCs and not neutrophils.

A 30-year-old male's blood work and biopsies indicate that he has proliferating osteoclasts that are producing large amounts of IgG. What is the man's most likely diagnosis? A)Acute myelogenous leukemia B)Multiple myeloma C)Acute lymphocytic leukemia D)Hodgkin lymphoma

B Feedback: One of the characteristics resulting from the proliferating osteoclasts in multiple myeloma is the unregulated production of a monoclonal antibody referred to as the M protein. In most cases, the M protein is either IgG or IgA. This phenomenon is not present in cases of CML, ALL, or Hodgkin lymphoma.

A 66-year-old female patient has presented to the emergency department because of several months of intermittently bloody stool that has recently become worse. The woman has since been diagnosed with a gastrointestinal bleed secondary to overuse of nonsteroidal anti-inflammatory drugs that she takes for her arthritis. The health care team would realize that which of the following situations is most likely to occur? A) The woman has depleted blood volume due to her ongoing blood loss. B) She will have iron deficiency anemia due to depletion of iron stores. C) The patient will be at risk for cardiovascular collapse or shock. D) She will have delayed reticulocyte release.

B Feedback: Ongoing blood loss is associated with iron deficiency anemia due to the depletion of iron stores. She is unlikely to have a depleted blood volume or be at risk for shock, situations more commonly associated with traumatic, sudden blood loss. There would not likely be a delay in the release of reticulocytes.

Which of the following patients would have a very poor response related to tissue regeneration of his or her injured area? A)A 21-day-old infant undergoing a diaphragmatic hernia repair B)A 54-year-old male who had a massive MI 4 days ago and came to the ED today for treatment C)A 73-year-old female who is undergoing lithotripsy for kidney stones D)A 33-year-old athlete undergoing surgery to repair a torn MCL in his right knee

B Feedback: Permanent or fixed cells cannot undergo mitotic division. The fixed cells include nerve cells, skeletal muscle cells, and cardiac muscle cells.

A student makes the statement to a colleague, "Blood plasma is essentially just a carrier for the formed elements like red blood cells and white blood cells." What would be the most accurate response to this statement? A) "That's not really true. Plasma is crucial in the immune and inflammatory responses." B) "Actually, plasma plays a significant role in nutrient and waste transport." C) "Not really. Plasma also contributes to the processes of protein synthesis and hematopoiesis." D) "Actually, plasma is integral to the proper function of the liver and maintenance of acid-base balance."

B Feedback: Plasma proteins are integral to transport. They are not noted to contribute to the inflammatory response, hematopoiesis, protein synthesis, or liver function.

A hospital laboratory technologist is analyzing the complete blood count (CBC) of a patient. Which of the following statements best reflects an aspect of the platelets that would constitute part of the CBC? A) Platelets originate with granulocyte colony-forming units (CFU). B) The half-life of a platelet is typically around 8 to 12 days. C) The á-granules of platelets contribute primarily to vasoconstriction. D) New platelets are released from the bone marrow into circulation.

B Feedback: Platelets' half-life is typically around 8 to 12 days. They originate from megakaryocytes, and ä-granules facilitate vasoconstriction. New platelets are released from the spleen into circulation.

The nurse knows which of the following statements best describes the characteristic trait of risckettsiae related to Rocky Mountain spotted fever? Rickettsiae A)are eukaryotic. B)have both RNA and DNA. C)have a distinct spiral-shaped morphology. D)are neither gram negative nor gram positive.

B Feedback: Rickettsiae combine characteristics of bacteria and viruses. Rickettsiae are prokaryotic but not spiral shaped. Rickettsiae are able to be classified by Gram stain like other microorganisms.

A patient who is recovering from burn injuries is discussing his prognosis with a physician. Which of the following teaching points about expectations for healing should the physician include? A) "Once your healing is complete, your skin will be just as strong as before your accident." B) "You may find that the scar is a bit smaller than the area of the wound." C) "You'll find that your new tissue is more elastic and fragile than the rest of your skin." D) "The final remodeling phase of healing may last up to 3 months in your case."

B Feedback: Scars are often smaller than the original area of the wound. There is nearly always an accompanying loss of strength and elasticity, and remodeling can take over 6 months.

Following a spider bite she received while camping, a 20-year-old female presented to the emergency department with rash, edema, and fever and was subsequently diagnosed with serum sickness. Which of the following statements best conveys the physiological rationale for the broad systemic effects of this event? A) The woman is experiencing diffuse tissue necrosis as a consequence of an Arthus reaction. B) Antigen-antibody complexes have been deposited in a variety of locations throughout the body. C) Antibody binding to specific target cell receptors is bringing about a change in cell function. D) Deposited antibodies are activating her complement system.

B Feedback: Serum sickness is characterized by the deposition of antigen-antibody complexes in blood vessels, joints, heart, and kidney tissue. The deposited complexes activate complement, increase vascular permeability, and recruit phagocytic cells, all of which can promote focal tissue damage and edema. Serum sickness is not synonymous with an Arthus reaction. Antibody binding to specific target cell receptors causing a change in cell function is characteristic of antibody-mediated cellular dysfunction. Serum sickness is not associated with the activation of the complement system.

A 70-year-old woman has received a diagnosis of chronic myelogenous leukemia (CML) after a clinical investigation sparked by the presence of leukocytosis in her routine blood work. What clinical course should her care provider tell her to expect? A)"You can expect your blood results, fatigue, and susceptibility to infection to gradually worsen over a few years." B)"You could remain the chronic stage of CML for several years before it accelerates and culminates in a crisis." C)"It's likely that this will give you chronic fatigue and malaise for the rest of your life, but that will probably be the extent of your symptoms." D)"Unfortunately, your leukemia will likely enter a crisis mode within a few weeks if we don't treat it immediately."

B Feedback: The clinical course of CML is commonly divided into three phases: (1) a chronic phase of variable length, (2) a short accelerated phase, and (3) a terminal blast crisis phase. Persons in the early chronic phase of CML generally are asymptomatic, but without effective treatment, most will enter the accelerated phase within 4 years.

A 48-year-old man who has been HIV positive for 6 years has just learned that he has been diagnosed with Kaposi sarcoma (KS). Which of the following facts most accurately conveys an aspect of his diagnosis? A) An opportunistic Epstein-Barr virus underlies the man's KS. B) He is likely to have lesions on his skin, mouth, or GI tract. C) Intense pain was probably his first manifestation of KS. D) Heterosexual contact most likely underlies his HIV and subsequent KS.

B Feedback: The lesions of KS can be found on the skin and in the oral cavity, gastrointestinal tract, and the lungs. More than 50% of people with skin lesions also have gastrointestinal lesions. It is linked with a herpes virus and can often be painless, especially in early stages. Men who have sex with men are at a higher risk of developing KS.

An 8-week-old boy has been recently diagnosed with a severe combined immunodeficiency (SCID). His parents have performed a significant amount of research on the Internet and have brought a large amount of material to discuss with their care provider. Which of the following statements best reflects an accurate understanding of their son's health situation? A) "We read that gene therapy could cure our son; we'd like you to look into that option." B) "Our son likely has a deficiency of B lymphocytes and can't produce antibodies." C) "We feel guilty, because dietary and environmental factors have been shown to contribute to SCID" D) "The antibodies that our son produces are mismatched to the infections that he was born with and encounters."

B Feedback: The pathophysiology of SCID involves normal B cells but a lack of antibody production because of inadequate T-cell help. Gene therapy is not yet a realistic treatment option, and the disease has a genetic basis. Antibodies are not incorrect but rather inadequate in number.

A client has a suspected infection by a particular microorganism in question that cannot be cultured. Which of the following processes listed below is most likely to result in an accurate diagnosis for the client? A) Observe directly for the presence or absence of specific antigens in the client's blood serum sample. B) Introduce cultured, marked antibodies to the client, and observe for a reaction with antigens in the client. C) Observe for a cytopathic effect on biopsy tissue samples from the client's mucosa. D) Release purified antigens into the client's circulation to observe whether the client has produced the relevant antibodies.

B Feedback: The process described is direct antigen detection, which can indirectly implicate a microorganism that is unable to be cultured. Answer D describes the same process in reverse, while answers A and C do not describe existing diagnostic processes.

While being on subcutaneous heparin injections for deep vein thrombosis during her latter pregnancy, a patient begins to experience major side effects. Her OB-GYN physician has called in a specialist who thinks that the patient is experiencing heparin-induced thrombocytopenia. The nurse should anticipate which of the following orders? A)Decrease the dose of heparin from 5000 units b.i.d to 3000 units b.i.d B)Immediately discontinue the heparin therapy C)Switch to Coumadin 2.5 mg once/day D)Infuse FFP stat

B Feedback: The treatment of HIT requires the immediate discontinuation of heparin therapy and the use of alternative anticoagulants to prevent thrombosis recurrence. Decreasing the dose will not stop HIT. Coumadin is contraindicated in pregnancy. FFP is not called for in this situation.

As part of her prenatal education, a 29-year-old woman who is pregnant with her first child is receiving teaching from her primary care provider. Which of the following statements by the woman reflects an accurate understanding of HIV transmission? A) "I know my baby is safe from HIV while in the womb, but the delivery will place him or her at real risk." B) "It's discouraging to know that my breast milk can pass on HIV to my baby." C) "I know it's possible, but it's comforting that the chances of my child contracting my HIV are actually very low." D) "I'm relieved to learn that a caesarean delivery will protect my baby from being born HIV positive."

B Feedback: Transmission from mother to infant is the most common way that children become infected with HIV. HIV may be transmitted from infected women to their offspring in utero, during labor and delivery, or through breast-feeding. Ninety percent of infected children acquired the virus from their mother. The risk of transmission of HIV from mother to infant is approximately 25%, with estimates ranging from 15% to 45%, depending on what country they live in.

A hospital client is receiving intravenous infusion of heparin for treatment of a pulmonary embolus. Which of the following phenomena is most likely to occur, resulting in the drug's therapeutic effect? A)Inhibition of vitamin K synthesis in the liver B)Suppression of fibrin formation C)Deactivation of the intrinsic clotting pathway D)Inhibition of ADP-induced platelet aggregation

B Feedback: Ultimately, heparin inhibits the clotting factors that mediate the formation of fibrin. It does not inhibit vitamin K synthesis, nor does it deactivate the intrinsic clotting pathway in particular. Heparin does not act on platelet plug formation.

Which of the following statements is an accurate descriptor of the role of viruses in human infections? A) Viruses have no genetic material of their own. B) Some viruses are capable of transforming normal host cells into malignant cells. C) Viruses are often implicated in cases of transmissible neurodegenerative diseases. D) Viruses require stimulation after a latent period before they are able to produce symptoms.

B Feedback: Viruses that are categorized as oncogenic are able to induce malignancy in host cells. Viruses have limited genetic material (either RNA or DNA), but no virus lacks genetic material. Transmissible neurodegenerative disease is associated with prions. Not all viruses include a latent period.

Which of the following procedures reduces the potential for infection primarily by addressing the portal of entry? A) Wiping down common areas with buffered bleach on a regular basis B) Wearing gloves when contact with blood or body fluids is anticipated C) Disposing of soiled clothing and bed linens in a dedicated receptacle D) Isolating patients who have antibiotic-resistant infections

B Feedback: Wearing gloves specifically blocks the portal of entry to the health care worker through the use of a physical barrier. Bleaching and cleaning, as well as disposing of soiled linen, eliminate the source of infection by killing microorganisms, and isolating patients similarly addresses a source by minimizing contact with uninfected persons.

A 60-year-old woman is suspected of having non-Hodgkin lymphoma (NHL). Which of the following aspects of her condition would help to rule out Hodgkin lymphoma? A)Her neoplasm originates in secondary lymphoid structures. B)The lymph nodes involved are located in a large number of locations in the lymphatic system. C)The presence of Reed-Sternberg cells has been confirmed. D)The woman complains of recent debilitating fatigue.

B Feedback: While NHLs tend to be multicentric, Hodgkin lymphoma tends to involve a single node or group of nodes. The presence of Reed-Sternberg cells would indicate Hodgkin lymphoma, and both NHL and Hodgkin lymphoma involve secondary lymphoid structures and would cause fatigue.

Health care team members know recently that an increased incidence of infections such as West Nile virus and SARS does not match with previously established patterns. Which of the following phenomena constitutes the most significant contributor to the spread of new diseases in the United States? A) Drug resistance by bacterial and protozoal infections B) Increased ease and frequency of individual and group international travel C) Genetic variation and mutation by microorganisms D) Decreased living standards and public health standards in urban areas

B Feedback: While drug resistance, lapses in public health, and genetic variation are all contributing factors in incidents of infection, the increase in new and global diseases is primarily attributed to the ease and frequency of individual and group international travel.

Which of the following processes would most likely be considered an anomaly during the cellular phase of inflammation? A)Platelet aggregation B)Vasoconstriction C)Migration of phagocytic white cells D)Macrophage activity

B Feedback: While vasoconstriction is a component of the immediate inflammatory response, the later cellular phase of inflammation is accompanied by vasodilation. Platelet aggregation, vasoconstriction, migration of phagocytic white cells, and macrophage activity are all associated with the cellular phase.

A patient has visited his physician because he found an enlarged lymph node along the mediastinal border. When questioned, the physician may be alerted to a possible diagnosis of Hodgkin lymphoma (HL) if the patient also displays: Select all that apply. A)yeast infection in the mouth. B)night sweats. C)unexplained pruritus. D)joint swelling. E)sore throat with pustules on tonsils.

B, C Feedback: Most people with HL present with painless enlargement of a single node or group of nodes. The initial lymph node involvement typically is above the level of the diaphragm. Mediastinal masses are frequent and discovered on routine chest x-ray. There may be complaints of chest discomfort with cough or dyspnea. Additional symptoms include fevers, chills, night sweats, and weight loss. Pruritus and intermittent fevers associated with night sweats are classic symptoms of HL. Yeast infections and joint swelling are not associated with HL. Sore throat with pustules on tonsils is associated with strep throat.

After years of going to different physicians with vague symptoms, a 55-year-old client with a history of Hodgkin disease has been diagnosed with a secondary immunodeficiency syndrome. The client asks the nurse what this means. The nurse knows from the following list of characteristics that secondary immunodeficiency disorders: Select all that apply. A) may be inherited as a sex-linked trait. B) usually develop later in life. C) may be a result of chemotherapy being used to treat a cancer. D) can result from frequent recurring Staphylococcus aureus infections. E) can occur in a chronic obstructive pulmonary disease patient taking corticosteroids daily.

B, C, E Feedback: Secondary immunodeficiency disorders develop later in life as a result of other pathophysiologic states such as malnutrition, disseminated cancers, infection of the cells of the immune system, and treatment with immunosuppressive drugs, such as chemotherapeutic agents. Primary disorders may be congenital or inherited as sex-linked, autosomal dominant, or autosomal recessive traits. Humoral (B-cell) immunodeficiencies are primarily associated with recurrent infections like Staphylococcus aureus.

Which of the following individuals would most likely be placed on highly active antiretroviral therapy (HAART) if he or she were not yet receiving the treatment? Select all that apply. A) A 35-year-old female sex trade worker who is HIV negative but who has a documented history of sharing needles for heroin use B) A 46-year-old male with long-standing HIV and a CD4+ count of 125 cells/mL C) A 16-year-old female who was diagnosed with HIV 2 days prior and is asymptomatic with normal CD4+ levels D) A 38-year-old woman who has a CD4+ count of 250 cells/mL and is keen to begin HAART E) Prophylactically to a health care worker who incurred a laceration from a scalpel used in surgery but has no abnormal lab results

B, D Feedback: All symptomatic patients should be treated with antiretroviral therapy. If the individual is asymptomatic, therapy is recommended for CD4+ cell counts less than or equal to 160/mL. For those who have a CD4+ cell count greater than 350 cells/mL, antiretroviral therapy is generally not recommended. For those whose CD4+ cell count is 160 to 350 cells/mL, then antiretroviral therapy should be considered, and a decision individualized to the patient should be made. HAART is not begun prophylactically in the absence of HIV.

A surgeon is explaining to the parents of a 6-year-old boy the rationale for the suggestion of removing the boy's spleen. Which of the following teaching points would be most accurate? A) "We believe that your son's spleen is causing the destruction of many of his blood platelets, putting him at a bleeding risk." B) "Your son's spleen is inappropriately filtering out the platelets from his blood and keeping them from normal circulation." C) "Your son's spleen is holding on to too many of his platelets, so they're not available for clotting." D) "We think that his spleen is inhibiting the production of platelets by his bone marrow."

C Feedback: A cause of thrombocytopenia is excessive sequestering of platelets by the spleen, necessitating splenectomy. The spleen would not be involved in destroying platelets, filtering them out from existing circulation, or inhibiting their production.

A 44-year-old male hospital client with a diagnosis of end-stage acquired immunodeficiency syndrome (AIDS) has been placed on neutropenic precautions that limit his interaction with visitors, staff, and other clients. What is the underlying rationale for these precautions? A)His antibody-mediated immunity is compromised by his low production of neutrophils. B)Neutropenia limits the ability of his CD4 helper cells to present antigens. C)Insufficient levels of neutrophils make him particularly susceptible to infections. D)Cyclic neutropenia limits his body's ability to fight various infections.

C Feedback: A decrease in the number of neutrophils places an individual at risk for infection. Neutrophils are not directly involved in the antibody-mediated immune process, and his neutropenia is infection related, not cyclic.

A 44-year-old Caucasian woman is being treated in an airport infirmary after she developed a painful, swollen leg during a transatlantic flight in economy class. The woman is suspected of having deep vein thrombosis (DVT) and is questioning the paramedics about why this might be the case, given that she has twice previously had similar experiences. Which of the following teaching points by the airport medical staff would be most appropriate? A) "A lot of Caucasian people have a genetic mutation that causes platelets to stick to their blood vessel walls." B) "There is a genetic disorder that causes many Caucasians to form more clots in their blood vessels." C) "A lot of Caucasians have an inherited inability to dissolve clots that form in their bodies." D) "Your doctor might be able to tell you if you've inherited a predisposition to bleeding in your veins."

C Feedback: All of the listed responses refer to the Leiden mutation, which is best characterized as an inhibition of normal clot dissolution due to factor V defects. It does not involve platelet aggregation or adhesion or excess bleeding. It is better characterized as decreased clot dissolution rather than increased clot formation.

Your ESRD patient is receiving 2 units of packed red blood cells for anemia (Hgb of 8.2). Twenty minutes into the first transfusion, the nurse observes the patient has a flushed face, hives over upper body trunk, and is complaining of pain in lower back. His vital signs include pulse rate of 110 and BP drop to 95/56. What is the nurse's priority action? A) Slow the rate of the blood infusion to 50 mL/hour. B) Document the assessment as the only action. C) Discontinue the transfusion and begin an infusion of normal saline. D) Recheck the type of blood infusing with the chart documentation of patient's blood type.

C Feedback: An immediate hemolytic reaction usually is caused by ABO incompatibility. The signs include flushing of the face, urticaria (hives), headache, pain in the lumbar area, chills, fever, chest pain, tachycardia, hypotension, and dyspnea. If any of these actions occur, the transfusion should be stopped immediately. Access to a vein should be maintained because it may be necessary to infuse IV solutions to ensure diuresis. Slowing the rate of the blood infusion will not correct this hemolytic reaction and will only worsen the patient's condition. Of course, documentation after the above interventions are performed is vital. Rechecking the blood type infusing will not stop the hemolytic reaction. After corrective actions/interventions are taken, the blood bag is returned to the blood bank for further testing.

A 23-year-old man has received a recent diagnosis of appendicitis following 24 hours of acute abdominal pain. The nurse providing care for the man is explaining that while unpleasant, the inflammation of his appendix is playing a role in his body's fight against the underlying infectious process. Which of the following teaching points should the nurse eliminate from his teaching to the patient? A)"Inflammation can help to remove the body tissue cells that have been damaged by infection." B)"Inflammation will start your body on the path to growing new, healthy tissue at the site of infection." C)"Inflammation helps your body to produce the right antibodies to fight the infection." D)"Inflammation ultimately aids in eliminating the initial cause of the cell injury in your appendix."

C Feedback: Antibody production is not a noted component of the inflammatory response. Removing damaged cells, generating new tissue, and eliminating the cause of cell injury are all documented components of the inflammatory response.

While undergoing a kidney transplant from a nonfamily member, the patient's transplanted kidney has just had the arterial clamps removed. The OR staff notice that the organ is turning purple with no urine output. When explaining to the family why they had to remove the donor kidney, the nurse will anticipate that the surgeon would likely include which statement? A) Obviously, there has been a mismatch during the human leukocyte antigen (HLA) testing. B) The circulating B and T lymphocytes are just doing their job. C) Hyperacute rejection occurs because antibodies against HLA antigens are deposited in vessels causing necrosis. D) Previous exposure to the HLA antigens is responsible for the high titers of complement fixing antibodies that cause the rejection.

C Feedback: Antibody-mediated rejection can be hyperacute, which occurs almost immediately after vascular reperfusion to graft tissue occurs. Performed antibodies against HLA antigens are deposited in the tissue endothelium and microvasculature where they activate the classic complement pathway causing tissue necrosis and graft injury.

When explaining what is occurring when their child has an acute bronchial asthma attack, the nurse will emphasize that which mediator is primarily responsible for the bronchial constriction? A)Tree pollen B)Mold dust C)Histamine D)T-lymphocyte proliferation

C Feedback: Asthma response begins within 5 to 30 minutes of exposure to an allergen. It is mediated by mast cell degranulation and the release of preformed and/or enzymatically activated mediators. These mediators include histamine, serotonin, and acetylcholine. Histamine is the most recognized mediator of type I hypersensitivity reactions and ultimately results in bronchial constriction.

A middle school student is scheduled to receive booster immunizations, and the father asks the nurse why the booster is necessary. What characteristic of the adaptive immune system listed below would provide the rationale for the nurse's response? A) Some antibodies require a repeat of the primary immune response. B) Some antibodies have a duration measured in months rather than years. C) A secondary response causes a sharp rise in antibody levels. D) Antigen receptors on CD4+ cells require multiple exposures separated by time.

C Feedback: Booster immunizations take advantage of the increase in antibodies that accompanies a repeat exposure. The primary immune response cannot be repeated, and antibodies survive beyond several months. Antigen receptors on CD4+ cells do not require multiple exposures.

A 4-year-old boy presents with a chronic cough and swollen lymph nodes. His records show that he has been given antibiotics several times in the past year with limited success, most recently for a liver abscess, and that he also has a recurring fungal skin condition. Which of the following is his most likely diagnosis? A)Selective IgA deficiency B)A deficiency in IgG2 subclass antibodies C)Chronic granulomatous disease D)Ataxia-telangiectasia

C Feedback: Chronic granulomatous disease, because it affects phagocytic function, increases susceptibility to soft tissue infections, particularly of the skin, lungs, lymph nodes, and liver. Selective IgA deficiency and deficient IgG2 subclass antibodies can predispose people to infection, but those infections respond readily to antibiotic treatment. Ataxia-telangiectasia can cause skin and liver problems, but its primary manifestations are ataxia and telangiectasia.

Utilizing the World Health Organization (WHO) framework of clinical categories for persons with acquired immunodeficiency syndrome (AIDS) over 15 years of age, a visitor to the United States goes to a city clinic complaining of diarrhea, weight loss of 20 lb, and feeling like he is running a temperature. These manifestations have been occurring for the past 5 weeks. The nurse would identify this patient to be in which clinical stage? A)Stage 1 B)Stage 2 C)Stage 3 D)Stage 4

C Feedback: Clinical stage 3 includes unexplained chronic diarrhea for greater than 1 month, persistent oral candidiasis, oral hairy leukoplakia, TB, neutropenia, anemia, and thrombocytopenia.

Which of the following patients is most likely to have impairments to the wound-healing process? A patient with A)chronic obstructive pulmonary disease. B)a diagnosis of multiple sclerosis and consequent impaired mobility. C)poorly controlled blood sugars with small blood vessel disease. D)congenital heart defects and anemia.

C Feedback: Diabetes mellitus is strongly associated with impaired wound healing. The other noted pathologies are less causative of deficiencies in the healing process.

An 81-year-old female patient in a subacute medical unit of a hospital has developed an oral Candida albicans infection. Which of the following phenomena would the patient's nurse suspect as a key contributing factor to her infection? A) The moist and temperature-suited oral environment of the client's mouth B) The ability of fungi to remain latent until the host reaches an immunocompromised state C) Antibiotic therapy that eliminated normal bacterial flora D) The airborne communicability of yeast and molds and subsequent inhalation

C Feedback: Elimination of bacterial flora that normally keeps colonizing fungi in check can induce the proliferation of fungi. The environment of the client's mouth provides a conducive environment but with an intact immune system is rarely a direct contributor to colonization. Latency is not a common trait of Candida albicans, and airborne transmission is not likely to be a causative factor.

A 62-year-old female with a diagnosis of acute and chronic renal failure secondary to diabetes mellitus is receiving her weekly injection of epoetin, a supplementary form of erythropoietin. Which of the following statements best captures the necessity of this medication? A) Erythropoietin is needed in order for stem cells to proliferate into committed erythroid precursors. B) Erythropoietin is necessary for the accurate sensation of hypoxia that stimulates erythropoiesis. C) Erythropoietin causes the erythrocyte colony-forming units to proliferate and mature. D) Erythropoietin facilitates the extrusion of the reticulocyte nucleus and the formation of true erythrocytes.

C Feedback: Erythropoietin acts primarily in later stages of erythropoiesis to induce the erythrocyte colony-forming units to proliferate and mature through the normoblast stage into reticulocytes and mature erythrocytes. It does not act directly on the stem cells, nor does it play a role in the sensation of hypoxia. Reticulocytes already lack a cell nucleus.

A 63-year-old woman has begun a diet that purports to minimize the quantity and effects of free radicals in her body. What physiological processes could best underlie her care provider's teaching about her new diet? A)Free radicals act as direct mediators in the inflammatory process. B)Free radicals inhibit the inflammatory response, limiting preadaptive response to infection. C)Free radicals increase cytokine expression and adhesion molecule levels, resulting in increased inflammation. D)Free radicals contribute to atherosclerosis and decreased immune response.

C Feedback: Free radicals are thought to bring about an inappropriate inflammatory response by increasing cytokines and numbers of adhesion molecule. They are not direct mediators of inflammation and are not associated with decreased immune response but rather inappropriate inflammation. Free radicals are not associated with inhibition of the inflammatory response.

Which of the following infectious agents listed below would be a site-specific pathogen and not spread throughout the body via the circulatory system? A patient diagnosed with A) B. burgdorferi caused by a tick bite. B) Salmonella typhi related to ingestion of contaminated food or water. C) Helicobacter pylori diagnosed after an endoscopic procedure. D) N. meningitidis infection in a child in a daycare setting.

C Feedback: H. pylori is an extreme example of a site-specific pathogen (localized infectious disease). The other three pathogens are called systemic pathogens because they can disseminate throughout the body via the circulatory system.

A patient diagnosed with H pylori asks the nurse, "How an infection can occur in the stomach since it is an acid environment?" The nurse responds, A) "We have many infectious agents that can live in an acidic environment with a pH more than 8.0." B) "H. pylori is a virus and is still being researched as to how it is able to survive in the stomach acids." C) "H. pylori produces an enzyme called urease that converts gastric juices into ammonia, which neutralizes the acidic stomach environment." D) "This parasite secretes an enzyme called coagulase, which protects the pathogen from the antibodies."

C Feedback: H. pylori, the infectious cause of gastritis and gastric ulcers, produces the urease enzyme on the outer cell wall. The urease converts gastric urea into ammonia, thus neutralizing the acidic environment of the stomach and allowing the organism to survive in this hostile environment.

A client with a diagnosis of sepsis has received intravenous immune globulin (IVIG) as a partial treatment. The nurse knows which of the following client responses listed below would best suggest an accurate understanding of IVIG treatment? A) "These antibodies in the solution have been collected from individuals who have successfully fought off the same infection." B) "The IVIG should help stimulate fever, inflammation, and tissue repair in the fight against the infection." C) "A big part of my IVIG treatment is actually stimulating and supplementing my immune system to do the work itself." D) "The main effect of IVIG is to cause my body to produce more white blood cells to fight infection."

C Feedback: IVIG involves infusion of pooled antibodies that supplement and stimulate the client's immune system to respond above and beyond its own capacities. It is not collected from individuals who have had similar infections. The primary effect of IVIG is not white blood cell production. Stimulation of fever, inflammation, and tissue repair are more closely associated with cytokine therapy.

A physician is explaining to a 40-year-old male patient the importance of completing his course of antibiotics for the treatment of tuberculosis. The physician explains the damage that could occur to lung tissue by Mycobacterium tuberculosis. Which of the following phenomena would underlie the physician's explanation? A) Tissue destruction results from neutrophil deactivation. B) Nonspecific macrophage activity leads to pulmonary tissue destruction and resulting hemoptysis. C) Macrophages form a capsule around the Mycobacterium tuberculosis bacteria, resulting in immune granulomas. D) Neutrophils are ineffective against the Mycobacterium tuberculosis antigens.

C Feedback: Immune granulomas are caused by insoluble particles (Mycobacterium tuberculosis) that are capable of inciting a cell-mediated immune response. Neutrophils are deactivated, self-destructive, nor ineffective in cases of tuberculosis.

Which of the following teaching points would be most appropriate with a client who has a recent diagnosis of von Willebrand disease? A) "It's important that you avoid trauma." B) "Your disease affects your platelet function rather than clot formation." C) "Make sure that you avoid taking aspirin." D) "Clotting factor VIII can help your body compensate for the difficulty in clotting."

C Feedback: No treatment other than the avoidance of aspirin is normally needed in the case of von Willebrand disease. Avoiding trauma and factor VIII therapy apply to hemophilia. Von Willebrand disease involves both the platelet and coagulation systems.

A group of nursing students were studying for their pathophysiology exam by quizzing each other about disorders of WBCs and lymphoid tissue. When asked what the first chromosomal abnormality that identified cancer was, one student correctly answered A)interleukin cells. B)BRCA-1. C)Philadelphia. D)PSA.

C Feedback: One of the more studied translocations is the Philadelphia chromosome, which was the first chromosomal abnormality identified in cancer. Cytokines or chemical messengers, such as interleukin (IL)-1, IL-4, IL-6, and interferon, act synergistically to support the functions of the growth factors. BRCA is a genetic test for breast cancer testing. BRCA mutations place the female at risk for developing cancers of the breast and ovary; PSA stands for prostate-specific antigen. It is a protein produced by prostate cells and is utilized to screen for prostate cancer.

A client who has a diagnosis of an autoimmune disease asks his nurse why it is that his immune system does not attack all of the cells that make up his body. Which of the following aspects of pathogen recognition in the innate immune system listed below would underlie the nurse's response? A) Normal host cells excrete inhibitory proteins that are detected by natural killer cells. B) Intraepithelial lymphocytes and natural killer cells possess specific, highly diverse receptors. C) Pattern recognition receptors (PRRs) ensure that cells are correctly identified. D) Leukocytes possess pathogen-associated molecular patterns (PAMPs)

C Feedback: PRRs recognize the structure of invaders and thus prevent activation by healthy somatic cells, though neither intraepithelial lymphocytes nor natural killer cells possess the high level of specification and diversity of receptors associated with the adaptive immune system. Host cells do not excrete inhibitory proteins, and PAMPs exist on pathogens, not on leukocytes.

A new nursing student is taking a tuberculin (TB) skin test as part of her preparation for beginning clinical placement in the hospital. The student is unclear of the rationale or physiology involved in this test. Which of the following is the correct explanation? A) The cell-mediated hypersensitivity associated with Mycobacterium tuberculosis remains detectable for several years. B) Formation of contact dermatitis lesions confirms prior TB contact. C) Previous TB exposure forms sensitized Th1 cells that are long-lived. D) This type of delayed-type hypersensitivity (DTH) is a response to latent Mycobacterium tuberculosis bacteria.

C Feedback: Sensitized Th1 cells form DTH response to introduced antigens. This is not an example of cell-mediated hypersensitivity or contact dermatitis, and it is not a response to latent Mycobacterium tuberculosis.

A 23-year-old HIV-positive woman in the United States with routinely low viral loads and robust CD4+ cell counts is planning to get pregnant. Which precaution would her care giver eliminate from her care? A) Offer her HAART that includes zidovudine B) Counsel her not to breast-feed C) Give her single-dose perinatal nevirapine D) Give the infant trimethoprim-sulfamethoxazole, starting at 4 to 6 weeks of age

C Feedback: Single-dose nevirapine is an appropriate alternative when zidovudine is not available. However, HAART-containing zidovudine is readily available in the United States. Avoiding breast-feeding will reduce the client's chances of transmitting HIV to her infant. Because the risk of transmission is not zero, prophylaxis with trimethoprim-sulfamethoxazole will protect her infant from PCP until its serostatus is known.

The nurse knows the cells primarily programmed to remove the invading organisms and remember the antigen to respond rapidly during the next exposure are A)CD4 and CD8 cells. B)natural killer (NK) cells and macrophages. C)T and B lymphocytes. D)white blood cells and platelets.

C Feedback: T and B lymphocytes are unique in that they are the only cells in the body capable of recognizing specific antigens present on the surfaces of microbial agents and other pathogens.

Two nursing students are attempting to differentiate between the presentations of immune thrombocytopenic purpura (ITP) and thrombotic thrombocytopenic purpura (TTP). Which of the students' following statements best captures an aspect of the two health problems? A) "Both diseases can result from inadequate production of thrombopoietin by megakaryocytes." B) "ITP can be either inherited or acquired, and if it's acquired, it involves an enzyme deficiency." C) "Both of them involve low platelet counts, but in TTP, there can be more, not less, hemostasis. D) "TTP can be treated with plasmapheresis, but ITP is best addressed with transfusion of fresh frozen plasma."

C Feedback: TTP is marked by sudden and severe thrombotic involvement. Neither disease has an etiology of low thrombopoietin production, and TTP, not ITP, is rooted in an enzyme deficiency. ITP is normally treated with corticosteroids and/or immunoglobulins.

A child has been diagnosed with thalassemia. Which of the following comorbidities may occur as a result of having thalassemia? A)Hypocoagulation B)Iron deficiency C)Splenomegaly D)Neutropenia

C Feedback: Thalassemia can result in enlargement of the spleen and liver due to increased hematopoiesis and red cell destruction. It is associated with thrombotic events, not hypocoagulation, as well as iron excess. Neutropenia is not associated with thalassemia.

When talking about the various types of granulocytes, which granule contains heparin, an anticoagulant? A)Neutrophils B)Eosinophils C)Basophils D)Lymphocytes

C Feedback: The basophils have granules that contain heparin, an anticoagulant; histamine, a vasodilator; and other mediators of inflammation.

A nurse practitioner is providing care for a client with low levels of the plasma protein gamma globulin. The nurse would recognize that the client is at risk of developing which of the following health problems? A)Jaundice B)Anemia C)Infections D)Blood clots

C Feedback: The gamma globulins, antibodies of the immune system, protect against infectious diseases. Because alpha globulin transports bilirubin, a defect in this plasma protein might lead to jaundice; a defect of beta globulin, which transports iron, could be associated with anemia; and a defect of fibrinogen could result in blood clots.

If an Rh-negative mother is giving birth to an Rh-positive infant, the nurse should be prepared to administer A)antihistamines like Benadryl. B)alpha interferon. C)Rh immune globulin. D)a monoclonal antibody like infliximab.

C Feedback: The injection of Rh immune globulin prevents sensitization in Rh-negative mothers who have given birth to Rh-positive infants if administered at 28 weeks' gestation and within 72 hours of delivery, abortion, or genetic amniocentesis. Antihistamines, alpha-interferon, or infliximab is not used in this situation.

A nurse is providing care for a 17-year-old boy who has experienced recurrent sinus and chest infections throughout his life and presently has enlarged tonsils and lymph nodes. Blood work indicated normal levels of B cells and free immunoglobulins but a lack of differentiation into normal plasma cells. The boy is currently receiving intravenous immunoglobulin (IVIG) therapy. What is the boy's most likely diagnosis? A)X-linked hypogammaglobulinemia B)Transient hypoglobulinemia C)Common variable immunodeficiency D)IgG subclass deficiency

C Feedback: The lack of a terminal differentiation from B cells into plasma cells is the hallmark of common variable immunodeficiency. Recurrent infections, enlarged lymph nodes and tonsils, and IVIG therapy are also commonly associated.

Staphylococcus aureus commonly found in the skin, nares, and other body sites of patients without any signs and symptoms of infection is known as which of the following conditions listed below? A)An opportunistic infection B)A parasitic infestation C)Bacterial colonization D)A saprophytic infection

C Feedback: The ongoing presence of bacteria in the body that do not cause infection is called colonization. Opportunistic infection by ordinarily nonpathogenic organisms can occur in immunosuppressed hosts. Parasitism is a condition in which the organism exists at the expense of, and without benefiting, the host. Saprophytes do not cause infection.

The spirochete leptospires is primarily transmitted to farmers by A)an airborne mechanism. B)exposure to spores in the environment. C)direct contact with infected animals. D)a mosquito bite.

C Feedback: The pathogenic leptospires infect a wide variety of wild and domestic animals. Infected animals shed the organisms into the environment through the urinary tract. Transmission to humans occurs by contact with infected animals or urine-contaminated surroundings. This spirochete is not transmitted by air, spores, or a fomite.

A couple who is expecting their first child has been advised by friends to consider harvesting umbilical cord blood in order to have a future source of stem cells. The couple has approached their caregiver with this request and is seeking clarification of exactly why stem cells are valuable and what they might expect to gain from harvesting it. How can their caregiver best respond to the couple's enquiry? Stem cells can A) "help correct autoimmune diseases and some congenital defects." B) "be used to regenerate damaged organs should the need ever arise." C) "be used as source of reserve cells for the entire blood production system." D) "help treat some cancers and anemias, but they must come from your child himself or herself."

C Feedback: The proliferative potential and self-renewal of stem cells make them a compensatory mechanism and reserve source for the entire hematopoietic system. While they could be of possible use in certain autoimmune conditions or in cases of organ failure, these statements do not capture their essence. Stem cells can also come from a histocompatible donor.

A client has been inhaling viruses periodically while on a cross-country flight. Which of the following situations listed below would most likely result in the stimulation of the client's T lymphocytes and adaptive immune system? A) Presentation of a foreign antigen by a familiar immunoglobulin B) Recognition of a foreign major histocompatibility complex (MHC) molecule C) Recognition of a foreign peptide bound to a self-major histocompatibility complex (MHC) molecule D) Cytokine stimulation of a T lymphocyte with macrophage or dendritic cell mediation

C Feedback: The stimulation of T cells requires the recognition of a foreign peptide bound to a self-major histocompatibility complex (MHC) molecule. Immunoglobulins do not play an antigen-presenting role, and foreign MHC molecules and cytokines do not stimulate the adaptive immune system.

A potential donor is angry at the personal nature of the questions about HIV risk factors that he is required to answer at a blood collection center and states that simple blood testing should suffice. How can the nurse at the center best respond? A) "There are some very uncommon subtypes of the HIV virus that are not detectable by current testing methods." B) "There's a chance that persons who are asymptomatic, but HIV positive can have their antibodies missed by serum testing." C) "There's a period shortly after someone is infected with HIV when blood tests might still be negative." D) "Even though blood tests are completely accurate, the high stakes of blood donation and transfusion mean that double measures are appropriate."

C Feedback: The time after infection and before seroconversion is known as the window period, during which HIV antibody screening may be negative. Potential donors are thus screened to identify potential risk factors. Undetectable subtypes of HIV do not exist, and individuals who are asymptomatic are still able to be accurately tested.

Which of the following phenomena would be least likely to result in activation of the complement system? A) Recognition of an antibody bound to the surface of a microbe B) Increase in tissue blood flow and capillary permeability, so fluids/proteins can leak into the area C) Activation of toll-like receptors (TLRs) on complement proteins D) Direct recognition of microbial proteins

C Feedback: Toll-like receptors are not associated with the complement system. The complement system may be activated by antibody recognition, mannose binding, or microbial protein recognition.

The physician knows the client G1P0 has correctly understood the prenatal education regarding sexually transmitted infection as evidenced by which of the following statements listed below? A) "Gonorrhea and chlamydia out of the various infectious agents pose the greatest risks of transmission from mother to child." B) "I know that my baby will need observation for HIV signs and symptoms in the weeks following my delivery." C) "My baby could become infected either across the placenta or during the birth itself." D) "If I receive prophylactic immunization, I will reduce my baby's chance of being born with an illness."

C Feedback: Vertical transmission may occur across the placenta in utero or during the birth event itself. Cytomegalovirus and HIV infections are the most common congenital infections. HIV signs and symptoms are not immediately apparent in the infant. Prophylactic immunization is not noted as a proven intervention for preventing vertical transmission.

A 39-year-old female with HIV infection has been characterized as a typical progressor by her care team and is experiencing an increase in her manifestations and health complaints as her CD4+ count declines. Which of the following health problems would her care team most likely attribute to a cause other than her HIV? A) Her recent diagnosis of bacterial pneumonia B) Her esophagitis that has been linked to herpes simplex infection C) Her decreased bone density and recent fractures D) Her increasing confusion and disorientation

C Feedback: While pneumonia, esophagitis, and cognitive deficits are all well-documented manifestations of HIV, changes in bone density are less likely to be a direct result of the virus.

A 12-year-old boy has contracted a bacterial infection at school, and his body has responded by increasing leukocyte production. Place the following components of white blood cell production in the correct chronological order. Use all the options. A) Myeloblast B) Promyelocyte C) Myeloid stem cell D) Metamyelocyte E) Neutrophil

C, A, B, D, E Feedback: Granulocytes like neutrophils derive from the myeloid stem cells. The immature precursor cells for each of the cell lines are called blast cells. Myeloblasts, which are the granulocytic precursor cells, have round to oval nuclei, with delicate chromatin and a blue to gray cytoplasm. During the next stage of development, the myeloblasts are transformed into promyelocytes with similar nuclei, but with a cytoplasm containing many primary granules. In the subsequent metamyelocyte stage, the nuclei distort and become arclike, producing the band developmental stage. Metamyelocytes mature into neutrophils.

While volunteering in an HIV clinic in a big city, the nurse notices a new mom and her baby (a 6-month-old male) in the waiting room. Upon assessing the infant for possible HIV infection, the nurse will be assessing for which of the following clinical manifestations of HIV infection? Select all that apply. A) Weighing him to determine if he is gaining 1.5 to 2 lb/month B) Observing to see if he can roll over from back to stomach C) Lack of coordination to play with toys/stuffed animals D) History of repeated episodes of bacterial pneumonia and ear infections E) Listlessness and poor eye contact

C, D, E Feedback: Children differ as to their clinical presentation of HIV infection when compared to adults. Failure to thrive (gain weight/height), CNS abnormalities (listlessness), and developmental delays are the most prominent primary manifestation of HIV infection in children. Answers A and B are normal growth and developmental tasks of a 6-month-old.

When talking about the lifespan of various blood components, the students should know that once a neutrophil moves into tissue, it lives approximately for how long? A)12 hours B)24 hours C)2 days D)4 days

D Feedback: After release from the marrow, the neutrophils spend only approximately 4 to 8 hours in the circulation before moving into the tissues. They survive in the tissues for approximately 4 to 5 days. They die in the tissues by discharging their phagocytic function.

The blood work of a 44-year-old male patient with a diagnosis of liver disease secondary to alcohol abuse indicates low levels of albumin. Which of the following phenomena would a clinician be most justified in anticipating? A)Impaired immune function B)Acid-base imbalances C)Impaired thermoregulation D)Fluid imbalances

D Feedback: Albumin plays a central role in the maintenance of osmotic pressure and fluid balance. Immune function, acid-base balance, and thermoregulation are not functions of albumin.

Which of the following types of pneumonia listed below is best characterized by an infective agent that produces sputum samples with a peptidoglycan cell wall, expresses endotoxins, replicates readily in broth and on agar, grows in clusters, has pili, and does not stain when exposed to crystal violet? A)Chlamydial B)Viral C)Mycoplasmal D)Bacterial

D Feedback: Although chlamydiae, viruses, and mycoplasmas all can cause pneumonia, only bacteria have all of these characteristics. Chlamydiae and viruses are obligate intracellular organisms and therefore would grow only in cell culture, and mycoplasmas lack the peptidoglycan cell wall typical of bacteria.

An oncologist has ordered a bone marrow biopsy for a client and is explaining the reasons for the test and what the client might expect during the test. Which of the following explanations best reflects an aspect of a bone marrow biopsy? A) "I'll take a sample of your bone marrow from your breastbone or your spine." B) "I will be harvesting a sample of your stem cells for examination." C) "I need a more accurate count of your blood components than normal blood work is able to provide." D) "I need to get samples of the types of blood cells that your body is producing."

D Feedback: Bone marrow biopsy can be used to detect abnormal cell production and gain samples. A sample would be taken most likely from the iliac crest, and the biopsy would not focus on pluripotent stem cells. Bone marrow biopsy is not used for simple quantification of blood cellular components.

he nurse knows that a drug in a category identified as a colony-stimulating factor (CSF) helps A) cells engulf and digest microbes that want to attach to cell membranes and destroy normal cell function. B) produce cells that will be the first responder cells to protect against cancer formation. C) stimulate the person's immune system so that he or she can kill his or her own cancer cells. D) stimulate bone marrow to produce large numbers of mature cells such as platelets and erythrocytes.

D Feedback: CSFs participate in hematopoiesis by stimulating bone marrow pluripotent stem and progenitor or precursor cells to produce large numbers of mature platelets, erythrocytes, lymphocytes, neutrophils, and monocytes.

Which of the following individuals would most likely possess normal plasma cell synthesis and fully differentiated myeloid and lymphoid cells? A)A 7-year-old boy with a diagnosis of acute lymphocytic leukemia (ALL) B)A 70-year-old male who has acute myelogenous leukemia (AML) C)A 58-year-old female with HIV and multiple myeloma D)A 78-year-old male who has been diagnosed with chronic lymphocytic leukemia (CLL)

D Feedback: Chronic leukemia is associated with proliferation of myeloid and lymphoid cells that are better differentiated than in cases of acute leukemia. Multiple myeloma would denote abnormalities of the plasma cells.

The nurse in the emergency department knows that clients exposed to Clostridium botulinum, an agent of bioterrorism, would likely be exhibiting which of the following clinical manifestations listed below? A) Blindness and respiratory distress B) Hemorrhage from all orifices resulting in signs of shock and coma C) Frothy, odiferous diarrhea and dehydration D) Muscle weakness in extremities eventually leading to paralysis of respiratory muscles

D Feedback: Clostridium botulinum causes neuromuscular paralysis and is listed as a category A agent.

A 1-year-old child who has experienced low platelet counts and bacterial susceptibility has been admitted to a pediatric medical unit of a hospital for treatment of Wiskott-Aldrich syndrome. The nurse who has admitted the child to the unit would anticipate which of the following short-term and longer-term treatment plans? A) Transfusion of clotting factors XII and XIII and serum albumin; splenectomy B) Neutropenic precautions; fresh frozen plasma transfusions; treatment of gastrointestinal symptoms C) Intravenous immunoglobulin (IVIG) treatment; thyroidectomy D) Treatment of eczema; management of bleeding; bone marrow transplant

D Feedback: Common interventions for Wiskott-Aldrich syndrome involve controlling eczema, managing bleeding due to low platelets, and ultimately bone marrow transplant. The other noted interventions are not associated with the treatment of Wiskott-Aldrich syndrome.

When educating a patient with a wound that is not healing, the nurse should stress which of the following dietary modifications to ward off some of the negative manifestations that can occur with inflammation? A) Increase the amount of calcium in the diet, especially drinking milk and eating cheese. B) This is the one time whereby you should eat more fat (both polyunsaturated and saturated), so you can absorb more fat soluble vitamins. C) Since there is a loss of plasma proteins, you should increase your intake of organ meats like liver. D) Increase your intake of oily fish and fish oil so that you will increase absorption of omega-3 polyunsaturated fatty acids.

D Feedback: Dietary modification of the inflammatory response through the use of omega-3 polyunsaturated fatty acids, specifically eicosapentaenoic acid and docosahexaenoic acid, which are present in oily fish and fish oil, may be effective in preventing some negative manifestations of inflammation.

In which of the following patients, would diagnostic investigations least likely reveal increased thrombopoietin production? A)An 81-year-old woman with diagnoses of rheumatoid arthritis and failure to thrive B)A 55-year-old man with dehydration secondary to Crohn disease C)A 66-year-old woman with a diagnosis of lung cancer with bone metastases D)A 21-year-old woman awaiting bone marrow transplant for myelogenous leukemia

D Feedback: Diseases such as myelogenous leukemia and other cases of primary thrombocytosis result in abnormalities in the thrombopoietin receptor and platelet binding. Cases of secondary thrombocytosis have an etiology rooted in increased thrombopoietin production. The common underlying causes of secondary thrombocytosis include tissue damage due to surgery, infection, cancer, and chronic inflammatory conditions such as rheumatoid arthritis and Crohn disease.

A 36-year-old woman with a diagnosis of antiphospholipid syndrome is receiving a scheduled checkup from her nurse practitioner. Which of the following teaching points would the nurse most likely prioritize? A) "It's important for you to do regular physical activity and maintain a healthy body weight." B) "Good nutrition and blood sugar control are important in your case." C) "You'll need to avoid taking nonsteroidal anti-inflammatory drugs when you have menstrual cramps." D) "You need to ensure your birth control pills don't contain estrogen."

D Feedback: Estrogen-containing birth control pills can predispose individuals with antiphospholipid syndrome to a thrombotic event. Exercise, nutrition, and blood sugar control are not particularly associated with management of antiphospholipid syndrome, and nonsteroidal anti-inflammatory drugs have no noted relevance to the disease.

Amniocentesis has suggested that a couple's first child will be born with sickle cell disease. The parents are unfamiliar with the health problem, and their caregiver is explaining the complexities. Which of the following statements by the parents would suggest a need for further teaching or clarification? A) "Our baby's red cells are prone to early destruction because of his or her weak membranes." B) "Not all of his or her red cells will be sickled, but low oxygen levels can cause them to become so." C) "Sickled cells can block his or her blood vessels, especially in the abdomen, chest, and bones." D) "Our son or daughter likely won't show the effects of sickling until he or she is school-aged because of the different hemoglobin in babies."

D Feedback: Fetal hemoglobin in the infant is replaced by 8 or 10 weeks of age, and manifestations of sickle cell disease can begin at this time. Answer choices A, B, and C all convey the aspects of sickle cell disease.

A community health nurse is conducting a class on the nutritional component for new mothers. Which of the following teaching points would be most justified? A)"Iron supplementation is not necessary provided you are breast-feeding your infant." B)"Be aware that cow's milk depletes your baby's supply of iron." C)"Your infant needs the same amount of iron as you but has far fewer sources for obtaining it." D)"If you choose to feed your baby with formula, ensure that it is iron fortified."

D Feedback: Formula and cereals for infants should be iron fortified to preclude iron deficiency anemia. Breast-feeding does not necessarily mitigate the need for iron supplementation, and cow's milk does not deplete existing iron stores but fails to provide sufficient levels of absorbable iron. Infants and children have significantly higher iron needs than do adults.

A hospital patient has a large, superficial wound on her elbow that was the result of shearing action when she was moved up in her bed. The patient's husband mentions that the wound looks infected and irritated because the wound bed is completely red. Which of the following responses would be inappropriate? A) "Even though it is red, it doesn't mean that the wound is infected." B) "The red areas show that there is enough circulation to facilitate healing." C) "Those are fresh blood vessels that are a sign of healthy healing." D) "A thin sheet of blood clotting is actually desirable and not a sign that your wife's wound is infected."

D Feedback: Granulation tissue indicates sufficient circulation and angiogenesis associated with healthy wound healing. Granulation tissue consists of new blood vessels, not clotted cellular components.

When looking at a granulocyte under a microscope, the anatomy student would describe it as a cell A)lacking granules. B)having a kidney-shaped nucleus. C)having no nuclei. D)shaped like a sphere with multilobar nuclei.

D Feedback: Granulocytes are spherical and have distinctive multilobar nuclei. They are all phagocytic cells that are identifiable because of their cytoplasmic granules. Answers A, B, and C are incorrect. Agranulocytes lack granular cytoplasm. Monocytes have a large amount of cytoplasm and a nucleus in the shape of a kidney. Erythrocytes lack a cell nucleus.

A client with a diagnosis of hemolytic anemia has gone to a community-based laboratory for follow-up blood work. The lab technician confirms with the client that hematocrit is one of the components of the blood work. The client replies, "I thought the point of the blood work was to see how many red blood cells I have today." How could the technician best respond to the client's statement? A) "This result will tell your care provider about the number of red blood cells in a given quantity of your blood plasma." B) "Your hematocrit measures the average size of your red blood cells and indirectly measures your oxygen-carrying capacity." C) "The result will indicate how many of your red blood cells are new and young and will indicate your body's production rate of red cells." D) "The hematocrit measures the mass that your red blood cells account for in a quantity of your blood."

D Feedback: Hematocrit measures the mass of erythrocytes in a given quantity of blood plasma. It does not measure the number of red cells, their size, or their production rate and age.

A new mother and father are upset that their 2-day-old infant is requiring phototherapy for hyperbilirubinemia. The pediatrician who has followed the infant since birth is explaining the multiplicity of factors that can contribute to high serum bilirubin levels in neonates. Which of the following factors would the physician be most likely to rule out as a contributor? A)The fact that the infant is being breast-fed B)Hypoxia C)Hepatic immaturity of the infant D)Transitioning of hemoglobin F (HbF) to hemoglobin A (HbA)

D Feedback: Hemoglobin transition from HbF to HbA is not associated with hyperbilirubinemia. Breast-feeding, hypoxia, and immaturity of the young liver can contribute to hyperbilirubinemia.

A 19-year-old intravenous drug user was exposed to the HIV 3 weeks ago and is experiencing a rapid proliferation in viral load. Which of the following statements best captures an aspect of the process of HIV replication that underlies this proliferation? A) Free HIV RNA is able to attach to the cell coat of CD4+ cells. B) The cytoplasm of CD4+ cells provides a protected environment for the replication of RNA by HIV. C) Expression of reverse transcriptase by CD4+ cells allows replication of HIV cells rather than new lymphocytes. D) HIV is able to change its RNA into DNA to allow for replication by CD4+ cells.

D Feedback: In order for the HIV to reproduce, it must change its RNA into DNA. It does this by using the reverse transcriptase enzyme. Reverse transcriptase makes a copy of the viral RNA and then in reverse makes another mirror-image copy. The result is double-stranded DNA that carries instructions for viral replication. HIV RNA does not directly attach to CD4+ cells, and RNA is not replicated by HIV itself in the CD4+ cytoplasm. Reverse transcriptase is not produced by CD4+ cells, and CD4+ cells do not directly produce new lymphocytes.

A 24-year-old woman has gone to the OB-GYN clinic for her first visit since she found out she was pregnant. The clinician tested her blood type along with the usual prenatal testing. On a follow-up visit, the woman was told that she is Rh negative. When asked what that means for her baby, the nurse explains that Rh-negative women lack RhD antigens on their erythrocytes but produce anti-D antibodies. As a result of this blood type, A) "If you and your baby have mismatched blood, it can invoke anaphylaxis in the baby." B) "If the types are incompatible, severe antibody-mediated inflammation occurs." C) "If blood types do not match, the baby's liver will produce extra cells to replace RBCs needed to oxygenate organs." D) "If the fetus is Rh positive, maternal anti-D antibodies can coat fetal RBCs resulting in severe anemia."

D Feedback: In utero, the development of erythroblastosis fetalis (Rh incompatibility) results when Rh-negative women produce anti-D antibodies. In Rh-positive fetus, maternal anti-D antibodies will coat fetal RBCs containing RhD, allowing them to be removed from the fetal circulation by macrophage- and monocyte-mediated phagocytosis (destroying RBCs). RBCs are produced in the bone marrow, not the liver; therefore, answer C is incorrect.

A patient presented to the emergency department of the hospital with a swollen, reddened, painful leg wound and has been diagnosed with methicillin-resistant Staphylococcus aureus (MRSA) cellulitis. The patient's physician has ordered a complete blood count and white cell differential. Which of the following blood components would the physician most likely anticipate to be elevated? A) Basophils B) Eosinophils C) Platelets D) Neutrophils

D Feedback: Increased neutrophils are associated with inflammation in general and bacterial infections in particular. Platelets play a role in inflammation, but their levels would not rise to the same extent as would neutrophils'. Eosinophils are not strongly associated with bacterial infection, and basophils would not increase to the same degree as neutrophils.

A geriatrician is following a number of clients on a subacute geriatric medical unit, some of whom require diagnostic blood work. Which of the following clients would be most likely to have an erythrocyte sedimentation rate (ESR) screening test ordered? An adult with A)a diagnosis of Alzheimer disease and depression. B)orthostatic hypotension and syncopal episodes. C)congestive heart failure. D)systemic lupus erythematosus.

D Feedback: Inflammatory diseases, such as systemic lupus erythematosus, are a common indicator for an ESR test. The other health problems noted are less commonly marked by inflammatory processes.

A 60-year-old male client with an acute viral infection is receiving interferon therapy. The physician is teaching the family of the client about the diverse actions of the treatment and the ways that it differs from other anti-infective therapies. Which of the following teaching points listed below should the physician least likely include? A) "Interferon can help your father's unaffected cells adjacent to his infected cells produce antiviral proteins that can stop the spread of the infection." B) "Interferon can help limit the replication of the virus that's affecting your father." C) "Interferon helps your father's body recognize infected cells more quickly." D) "Interferon can bolster your father's immune system through the stimulation of natural killer cells that attack viruses."

D Feedback: Interferons can activate macrophages in the fight against viral invaders, but they are not noted to stimulate the action of natural killer cells. Answers A, B, and C all capture elements of the action of interferons.

A 22-year-old female who adheres to a vegan diet has been diagnosed with iron deficiency anemia. Which of the following components of her diagnostic blood work would be most likely to necessitate further investigation? A)Decreased mean corpuscular volume (MCV) B)Decreased hemoglobin and hematocrit C)Microcytic, hypochromic red cells D)Decreased erythropoietin levels

D Feedback: It would be unusual to note decreased levels of erythropoietin concurrent with a diagnosis of anemia. Decreased MCV, hematocrit, and hemoglobin are congruent with the diagnosis, as are microcytic, hypochromic erythrocytes.

A 79-year-old female resident of an assisted living facility receives care from a community nurse on a regular basis for treatment of a chronic venous leg ulcer. Which of the following factors would the nurse be most justified in ruling out as a contributing factor to the client's impaired wound healing? A)A lower skin collagen content than in younger adults B)Decreased fibroblast synthesis C)Slow reepithelialization D)Decreased antibody levels

D Feedback: Older adults do not normally have diminished antibody levels. Low collagen levels, decreased fibroblast activity, and slow reepithelialization are common impediments to wound healing in the elderly.

Which of the following situations can best be characterized as an example of passive immunity? A) A 6-month-old infant receives his scheduled immunization against measles, mumps, and rubella. B) A 9-year-old boy is immune to chicken pox after enduring the infection before 1 year. C) An 8-year-old girl recovers from a respiratory infection after intravenous antibiotic treatment. D) A 6-week-old infant receives antibodies from his mother's breast milk.

D Feedback: Passive immunity involves the transfer of antibodies from an outside source, such as those from breast milk. Immunization and recovery from illness involve active immunity.

The school nurse knows several children with hemophilia A. After recess, one student with hemophilia comes to the school nurse complaining of pain in his knee from falling on the playground. The nurse notes there is swelling in the knee and pain on palpation. The nurse should A) administer some NSAIDs to relieve the pain. B) wrap the knee in an ace bandage for compression. C) apply some warm compresses to the knee. D) notify parents to pick up the child and possibly administer factor VIII.

D Feedback: Prevention of trauma is important. ASA and other NSAIDs that affect platelet function should be avoided. Factor VIII replacement therapy administered at home has reduced the typical musculoskeletal damage. Wrapping with a bandage will not prevent damage. Warm compression will extend the bleed.

The nurse knows which of the following statements below is appropriate to be included in an education session for a 21-year-old male with a diagnosis of malaria? A) "Your infection likely began with the introduction of fertilized protozoal ova from a mosquito." B) "The protozoa responsible have hijacked the genetic material of your cells in order to reproduce." C) "You are very tired because the pathogens are utilizing the ATP that your own cells need." D) "The infectious organisms are considered tiny, single-celled animals, given their complete eukaryotic machinery."

D Feedback: Protozoa possess full eukaryotic machinery including organelles and a nucleus. Ova are associated with helminthes, and protozoa do not utilize the host's genetic material for reproduction. Appropriation of ATP is associated with chlamydiae.

A 40-year-old woman who experiences severe seasonal allergies has been referred by her family physician to an allergist for weekly allergy injections. The woman is confused as to why repeated exposure to substances that set off her allergies would ultimately benefit her. Which of the following phenomena best captures the rationale for allergy desensitization therapy? A) Repeated exposure to offending allergens binds the basophils and mast cells that mediate the allergic response. B) Allergens in large, regular quantities overwhelm the IgE antibodies that mediate the allergic response. C) Repeated exposure stimulates adrenal production of epinephrine, mitigating the allergic response. D) Injections of allergens simulate production of IgG, combining with the antigens to prevent activation of IgE antibodies.

D Feedback: Repeated exposure to allergens causes an increase in IgG, which binds with antigens before they can stimulate IgE. It does not bind mast cells or basophils, nor does it overwhelm the IgE antibodies to stimulate epinephrine production.

A 37-year-old male with HIV who has recently become symptomatic has begun highly active antiretroviral therapy (HAART). Among the numerous medications that the man now regularly takes are several that inhibit the change of HIV RNA to DNA in a CD4+ cell. Which of the following classes of medications addresses this component of the HIV replication cycle? A)Entry inhibitors B)Protease inhibitors C)Integrase inhibitors D)Non-nucleoside reverse transcriptase inhibitors

D Feedback: Reverse transcriptase inhibitors inhibit HIV replication by acting on the enzyme reverse transcriptase. Non-nucleotide reverse transcriptase inhibitors block the copying of RNA into DNA. Entry inhibitors, protease inhibitors, and integrase inhibitors do not address this aspect of the HIV replication cycle.

A client has been identified as having an excess of macrophage inhibitory factor, causing the client to have inhibited movement and activity of macrophages. Which of the following processes listed below would the health care team member expect to remain unaffected? A) Amplification of the immune response B) Destruction of virus-infected or tumor cells C) Initiation of adaptive immunity D) Specificity and memory of the immune response

D Feedback: Specificity and memory are the defining characteristics of the adaptive immune system, and macrophages do not perform this particular role. Amplification of the immune response, destruction of virus-infected or tumor cells, and initiation of adaptive immunity are all components of macrophage activity.

A medical student is familiarizing herself with recent overnight admissions to an acute medical unit of a university hospital. Which of the following patients would the student recognize as least likely to have a diagnosis of antiphospholipid syndrome in his or her medical history? A) A 66-year-old obese male with left-sided hemiplegia secondary to a cerebrovascular accident B) A 90-year-old female resident of a long-term care facility who has been experiencing transient ischemic attacks C) A 30-year-old female with a diagnosis of left leg deep vein thrombosis and a pulmonary embolism D) A 21-year-old male with a diagnosis of cellulitis and suspected endocarditis secondary to intravenous drug use

D Feedback: Stroke, transient ischemic attacks, deep vein thrombosis, and pulmonary emboli are all common manifestations of the hypercoagulability associated with antiphospholipid syndrome. Cellulitis, endocarditis, and other infectious processes are less likely to be correlated with antiphospholipid syndrome.

A patient has been diagnosed with anemia. The physician suspects an immune hemolytic anemia and orders a Coombs test. The patient asks the nurse what this test will tell the doctor. The nurse replies, A) "They will wash your RBCs and then mix the cells with a reagent to see if they clump together." B) "They will look at your RBCs under a microscope to see if they have an irregular shape (poikilocytosis)." C) "They will be looking to see if you have enough ferritin in your blood." D) "They are looking for the presence of antibody or complement on the surface of the RBC."

D Feedback: The Coombs test is used to diagnose immune hemolytic anemias. It detects the presence of antibody or complement on the surface of the red cell. Answer choice A refers to direct antiglobulin test (DAT). Answer choice B refers to blood smear test. Answer choice C refers to iron stores test.

A 40-year-old male client is shocked to receive a diagnosis of mature B-cell lymphoma and is doing research on his diagnosis on the Internet. Which of the following statements that he reads on various Web sites is most reliable? A)"Like most forms of Hodgkin lymphoma, mature B-cell lymphoma often requires radiation treatment." B)"Doctors are able to diagnose mature B-cell lymphoma by the presence of Reed-Sternberg cells." C)"Unlike many other lymphomas, mature B-cell lymphoma is often self-limiting, and treatment is focused on symptoms." D)"The lymph nodes are usually affected, and often the spleen and bone marrow."

D Feedback: The NHL subtypes of mature B-cell lymphoma may affect the lymph nodes, spleen, or bone marrow. It is not a type of Hodgkin lymphoma, and Reed-Sternberg cells would indicate Hodgkin lymphoma. It is not self-limiting and necessitates active treatment.

A client with a gastrointestinal bleed secondary to alcohol abuse and a hemoglobin level of 5.8 g/dL has been ordered a transfusion of packed red blood cells. The client possesses type B antibodies but lacks type D antigens on his red cells. Transfusion of which of the following blood types would be least likely to produce a transfusion reaction? A)B- B)B+ C)A+ D)A-

D Feedback: The client's blood type is A- and would necessitate A- or O- donor blood. Other types would induce a transfusion reaction.

A 53-year-old female hospital patient has received a kidney transplant following renal failure secondary to hypertension. As part of the teaching while she was on the organ wait list, she was made aware that she would need to take antirejection drugs for the rest of her life. Which aspect of the immune system underlies this necessity? A) The lack of identifiable major histocompatibility complex (MHC) molecules will stimulate the innate immune response. B) Donor organ antibodies will be identified as foreign and stimulate an immune response. C) Antirejection drugs will stimulate the production of familiar MHC molecules. D) MHC molecules will never develop in the cells of the donor organ, and effector cells will be continually stimulated.

D Feedback: The lack of familiar MHC molecules will stimulate an immune response by effector cells in the absence of antirejection drugs. An innate immune response is not central to the response, but rather the adaptive immune system. Lack of known MHC molecules, not foreign antibodies, accounts for the immune response, and familiar MHC molecules will not be produced by the donor kidney cells.

A 5th grade elementary student asks the school nurse how much blood is in an entire body. The nurse should respond that the average grown-up adult has A)2 to 4 cups of blood in his or her body. B)3 pints of blood in total. C)3 to 4 quarts of blood in his or her body. D)5 to 6 L of blood throughout his or her body.

D Feedback: The total volume of blood in the average adult is about 5 to 6 L, and it circulates throughout the body within the confines of the circulatory system.

A person who has been diagnosed with HIV infection 12 years ago and still has a CD4+ cell count of 800 cells/µL and a low viral load is considered clinical to be a A)rapid progressor. B)typical progressor. C)slow progressor. D)long-term nonprogressor.

D Feedback: There is a subset of slow progressors: the long-term nonprogressors, who account for 1% of all HIV infections. These people have been infected for at least 8 years, are antiretroviral naive, have high CD4+ cell counts, and usually have very low viral loads. They are being investigated to determine how they maintain viral suppression of HIV.

Members of an AIDS support group who have more advanced cases are sharing some of their recent health problems with a member who has just been diagnosed. Which of the member's statements is most accurate? A) "One of the scariest things out there now is the huge increase in drug-resistant tuberculosis." B) "The eradication of Pneumocystis jiroveci pneumonia (PCP) has helped extend the life expectancy of a lot of persons living with AIDS." C) "Those of us with HIV are so much more prone to loss of vision and hearing." D) "As people with HIV live longer, most of us are eventually succumbing to the cancers that are associated with HIV."

D Feedback: There is an increased risk of AIDS-associated cancers as persons with age live longer. Drug resistance in tuberculosis is on the decline in recent years, and PCP has not been eradicated. Sensory loss is not a noted HIV-related manifestation.

A client has been diagnosed with herpes simplex virus. The client states that, "modern medicine produces more and more antivirals every year, and so the treatment should be simple." Which of the following statements listed below is the best response? A) "The recent rise of drug resistance has significantly hampered the elimination of viruses." B) "The cell coat of viruses is particularly resilient to the available synthetic antivirals." C) "The use of antivirals is severely limited by the unwanted adverse effects that they cause." D) "Treatment options for viruses are often limited because what destroys viruses often damages your own body cells."

D Feedback: Viruses are difficult to treat because interference with their replication often requires interference with the body's cell replication processes. Although they do exist with antiviral treatments, drug resistance and side effects are phenomena more closely associated with antibacterials. Antivirals act upon DNA or RNA synthesis, not the cell wall.

A 16-year-old female has been brought to her primary care physician by her mother due to the girl's persistent sore throat and malaise. Which of the following facts revealed in the girl's history and examination would lead the physician to rule out infectious mononucleosis? A)The girl has a temperature of 38.1°C (100.6°F) and has enlarged lymph nodes. B)Her liver and spleen are both enlarged. C)Blood work reveals an increased white blood cell count. D)Chest auscultation reveals crackles in her lower lung fields bilaterally.

D Feedback: While fever, enlarged lymph nodes, splenomegaly, hepatomegaly, and leukocytosis would suggest mononucleosis, adventitious lung sounds are not associated with the disease.

A 53-year-old man presents with inability to concentrate, itching in his fingers and toes, elevated blood pressure, and unexplained weight loss. He is diagnosed with primary polycythemia. What will be the primary goal of his treatment? A)To control his hypertension B)To increase the amount of oxygen distributed by his red blood cells C)To reduce the mean size of his red cells D)To reduce the viscosity of his blood

D Feedback: While hypertension may accompany polycythemia vera, the primary goal of treatment is to control the increase in blood viscosity that accompanies the disease. Polycythemia vera is not associated with increased corpuscular volume, and oxygen distribution is not a priority problem.

A 16-year-old girl has broken her arm while snowboarding. When she arrives at hospital, she is shocked at the amount of swelling at the injury site. Which of the following statements best explains the physiological rationale for her swelling? A) Migration and proliferation of mast cells, neutrophils, and platelets to the injury site occupy an increased volume of tissue. B) Potent vasodilation increases the total volume of vascular space at the site of inflammation. C) Osmotic flow of plasma into the intravascular space causes increased blood volume and interstitial fluid. D) Loss of plasma proteins causes an increase in interstitial osmotic pressure.

Feedback: Swelling is the result of plasma proteins leaving the interstitial space, resulting in increased osmotic pressure of interstitial fluid and movement of fluid into tissues. Blood components, vasodilation, nor increased intravascular volume accounts for swelling.


Kaugnay na mga set ng pag-aaral

Entrepreneurship Exam 3 (Chapters 10-14)

View Set

History Exam: Ch. 6- Scientific Revolution and the Enlightenment The statements below best describe which period? * Captionless Image

View Set

Chapter 7 Exam: Group Life Insurance

View Set

A&P Muscular System Lecture EXAM #3

View Set

HRMT 101 Week 2 - Basic Principles of APA Style & Formatting

View Set

NU141- Chapter 55 Drugs Acting on the Lower Respiratory Tract

View Set

Pharmacology ATI Oral Medications

View Set

IT 467 midterm - SQL and firewall zone policy

View Set

CH9: Disability Income and Related Insurance

View Set